IASbaba's Flagship Course: Integrated Learning Programme (ILP) - 2024  Read Details

Posts

Daily Prelims CA Quiz

UPSC Quiz - 2020 : IASbaba's Daily Current Affairs Quiz 10th Oct 2020

For Previous Daily Quiz (ARCHIVES) - CLICK HERE The Current Affairs questions are based on sources like ‘The Hindu’, ‘Indian Express’ and ‘PIB’, which are very important sources for UPSC Prelims Exam. The questions are focused on both the concepts and facts. The topics covered here are generally different from what is being covered under ‘Daily Current Affairs/Daily News Analysis (DNA) and Daily Static Quiz’ to avoid duplication. The questions would be published from Monday to Saturday before 2 PM. One should not spend more than 10 minutes on this initiative. This is a part of our recently launched, NEW INITIATIVE IASbaba’s INTEGRATED REVISION PLAN (IRP) 2020 – Road Map for the next 100 Days! FREE INITIATIVE! We will make sure, in the next 4 months not a single day is wasted. All your energies are channelized in the right direction. Trust us! This will make a huge difference in your results this time, provided that you follow this plan sincerely every day without fail. Gear up and Make the Best Use of this initiative. Do remember that, “the difference between Ordinary and EXTRA-Ordinary is PRACTICE!!” To Know More about the Initiative -> CLICK HERE SCHEDULE/DETAILED PLAN – > CLICK HERE Important Note: Don't forget to post your marks in the comment section. Also, let us know if you enjoyed today's test :) After completing the 5 questions, click on 'View Questions' to check your score, time taken and solutions. To take the Test  - Click Here

Important Articles

[NEW BATCH] IASbaba’s PROGRAMS/COURSES –UPSC 2021: E-CLP, ILP, AIPTS, TLP – Prelims, Mains Integrated Programs and Test Series – ADMISSIONS OPEN!

Hello Everyone, We are happy to announce the second batch of Integrated Learning Programme -2021, All India Prelims Test Series 2021, e-classroom Learning Programme 2021 and Think Learn and Perform (TLP)-2021. Essence of these Programmes: e-CLP 2021 - Online Classroom Program for fresher’s. ILP 2021- Integrated (Prelims + Mains + Interview) Program for Fresher’s/Veterans. It includes both Prelims and Mains Test Series and Evaluation and Mentorship. AIPTS 2021 - Exclusive Prelims Test Series TLP Connect 2021 - Integrated (Prelims + Mains + Interview) Test Series Based Mentorship Program for those who have completed their syllabus atleast once or twice/who have given Mains/Interview. TLP+ 2021 - Exclusive Mains Test Series and Mentorship In UPSC 2019, 150+ (from ILP and TLP ONLY) students from IASbaba got an Interview call. It’s a Proud moment!  All Credit goes to the efforts of our students put in the right direction!  100+ Ranks UPSC CSE 2019 Topper’s from IASbaba – 5 in Top 20 Ranks makes it even more Special! -> CLICK HERE We are also thankful and overwhelmed by the response, thank you note and feedback from our previous years’ students after UPSC Prelims 2020. Getting such feedback and emails/calls always excite us. You should go through this PDF - Detailed Explanation of UPSC Prelims 2020 to understand and assess the quality. Do go through the PDF sincerely. Let us go through some unique questions asked by UPSC in Prelims 2020: Q.) With reference to cultural history of India consider the following pairs: Parivrajaka Renunciant and Wanderer Shramana Priest with a high status Upasaka Lay follower of buddhism Which of the pairs given above are correctly matched? 1 and 2 only 1 and 3 only 2 and 3 only 1,2 and 3 Solution (b) REFERENCE: ILP Value Add Notes-Ancient India: All three were covered in VAN- Direct Hit Q.) With reference to the cultural history of India, which one of the following is the correct description of the term ‘paramitas’? The earliest Dharmashastra texts written in aphoristic (sutra) style. Philosophical schools that did not accept the authority of Vedas. Perfections whose attainment led to the Bodhisattva path Powerful merchant guilds of early medieval south India. Solution (c) REFERENCE: ILP Value Add Notes-Ancient India Q.) With reference to the history of India, “Ulgulan” or the Great Tumult is the description of which of the following events? The Revolt of 1857 The Mappila rebellion of 1921 The Indigo revolt of 1859 – 60 Birsa Munda’s revolt of 1899-1900 Solution (d) REFERENCE: ILP Value Add Notes-Ancient India Q.) In the context of recent advances in human reproductive technology, ‘Pronuclear Transfer” is used for Fertilization of egg in vitro by the donor sperm Genetic modification of sperm producing cells Development of stem cells into functional embryos Prevention of mitochondrial diseases in offspring Solution (d) REFERENCE: Prelims Exclusive Programme (PEP)-Handout: Value Add Notes Like this, we have a lot of direct hits in recent UPSC Prelims 2020. You can cross-check this in the given PDF above. UPSC is known to go against all expectations, predictions and pre-conceived notions of the candidates. That’s why most of the students who appear for the exam just get bamboozled by the question paper. Some feel that it is too easy and they were preparing for a much more difficult paper and some feel that they were preparing in a different dimension all together. Only a few are able to hit the bull’s eye. We are telling this so that if you have missed the mark this year, don’t beat yourself too hard. This year was full of turmoil and we are all facing situations that we had seen only in some Hollywood fiction movies. The pandemic, the lockdown and too much stress in the environment. Under these circumstances even the tougher ones can be thrown off the track. However, this is not the time to sulk, rather to mend your previous mistakes and look to the future. You knew it is going to be a difficult and long journey. So rather than wasting your time in thinking about what you could have done, invest your time in what you can do now. Start with positivity and have self-belief. At this point we would like to inform you that we will be starting a new batch for (Integrated Learning Program (ILP), All India Prelims Test Series (AIPTS) and e – Classroom Learning program (e – CLP). We were getting a lot of requests from the students that they couldn’t join the program earlier as prelims was delayed. Now is the opportunity for you to get enrolled and get the right guidance under the best mentors/teachers. Please Note: Both TLP Connect and TLP+ Programs are available OFFLINE & ONLINE. But given the present conditions you can take the programme ONLINE and when conditions are back to normal you can change to OFFLINE Mode. 1. Integrated Learning Programme- 2021 Integrated (Prelims + Mains + Interview) Program for Fresher’s/Veterans. It includes both Prelims and Mains Test Series and Evaluation and Mentorship. In the last five years, ILP has been bang on target with a hit ratio of more than 65%, a figure remarkably high keeping in mind the unpredictable nature of UPSC. With a knack of getting it right ILP has become a reliable name. The new batch of ILP 2021 will be starting from 12th October 2020. Read the complete details of ILP-2021 -> Click Here It has three different programmes. ILP Basic ILP Plus ILP Connect Download the New Schedule -> Click Here Payment Links ILP Basic Price: Rs.12,000 + 18 % (GST) = Rs.14,160/- Make Payment - English ILP Plus Price: Rs.15,000 + 18 % (GST) = Rs.17,760/- Make Payment - English ILP Connect Plus Price: Rs.20,000 + 18 % (GST) = Rs.23,600/- Make Payment - English 2. All India Prelims Test Series-2021 Exclusive Prelims Test Series. IASbaba is known for its quality coverage and it is a testimony that we have been able to HIT the bull’s eye in Prelims and Mains both over the period of the last 5 years. The closeness and exact repetition of Questions from our Tests needs no mention. This is acknowledged by many Toppers every year. Well, apart from the fact that many toppers have recommended IASbaba for our quality, the overall results are a testimony of our meticulous planning and dedication towards students. We try to match the standards set by UPSC in our test series by keeping the questions of mixed difficulty level i.e. easy, moderate, difficult, and very difficult. We have two different courses under AIPTS 2021. AIPTS 2021 AIPTS Plus 2021 The new batch of ILP 2021 will be starting from 21st October 2020. Read the complete details of AIPTS-2021 -> Click Here It has two different programmes. AIPTS 2021 AIPTS PLUS 2021 Download the New Schedule -> Click Here Payment Links AIPTS Price: Rs.5,999 + 18 % (GST) = Rs.7,080/- Make Payment - English AIPTS Plus Price: Rs.7,999 + 18 % (GST) = Rs.9,440/- Make Payment - English 3. e-Learning Classroom Programme (e-CLP)-2021 If you are a Fresher and needs classroom programme with conceptual clarity, this programme is for you. Hybrid Model of Classes Integrated & the Most Comprehensive CLASSROOM Program Focus on Fundamentals through Strategy/Approach Classes Focus on Conceptual Understanding & Value Add Notes Daily Prelims and Mains Tests/Assignment and Fortnightly Full-Length tests Connecting the Dots – Sessions by Experts & Toppers Live Doubt Clearing Sessions Subject-wise Mentorship by Expert The new batch of ILP 2021 will be starting from 21st October 2020. Read the complete details of e-CLP-2021 -> Click Here Download the New Schedule -> Click Here Payment Links e-CLP​ Rs.61,000 + Rs.10,980(18% GST) = Rs. 71,980​ Pay Full Payment e-CLP 1st Installment​ Rs.24,400 +  Rs.4,392(18% GST) = Rs. 28,792 Pay 1st Installment 4. TLP CONNECT 2021 (Prelims + Mains+ Interview) INTEGRATED Mentorship Based Programme and TLP PLUS (+) 2021 MAINS Mentorship Based Programme (OFFLINE and ONLINE) TLP Connect 2021 Integrated (Prelims + Mains + Interview) Test Series Based Mentorship Program for those who have completed their syllabus at least once or twice/who have given Mains/Interview. In UPSC 2019, 150+ (from ILP and TLP ONLY) students from IASbaba got an Interview call. It’s a Proud moment!  All Credit goes to the efforts of our students put in the right direction! 100+ Ranks UPSC CSE 2019 Topper’s from IASbaba – 5 in Top 20 Ranks makes it even more Special! -> CLICK HERE For the past 5 years, TLP has become synonymous with quality and effectiveness. The testimony and recommendation of toppers and a hit ratio of more than 82% in Mains (in the last 4 Years) speak volumes about the quality of questions that IASbaba frames. In TLP, you will get a mix of thought provoking, opinion oriented, highly analytical and most probable questions apart from few direct questions to prepare you for any scenario that UPSC can throw at you. You can see it yourself here: 2016, 2017, 2018, 2019 Going by the success of last year, we are re-launching both the versions of Think Learn Perform (TLP) Programme. Read the details below: TLP PLUS (+) 2021 (Available in BOTH OFFLINE and ONLINE) TLP CONNECT 2021 (Available in BOTH OFFLINE and ONLINE) Topper's Testimonials Previous Next Comparison Chart of all the Programmes-2021 Offline Payment Details Bank Details BANK Name CANARA BANK BANK BRANCH BANGALORE KLE SOCIETY ACCOUNT NAME IASBABA ACCOUNT NUMBER 8418201003836 IFSC CODE CNRB0008418 ACCOUNT TYPE CURRENT ACCOUNT  AFTER MAKING PAYMENT ONLINE You will receive a Receipt/Mail with Login Details. Do not worry it may take 2-3 hours time since it is automated. You can always mail for any technical issue on ilp@iasbaba.com   FOR ANY QUERIES (Related to Programmes of 2021) You can reach us on Email id's: eclp@iasbaba.com ilp@iasbaba.com support@iasbaba.com tlp@iasbaba.com Mobile No's: 9986193413 / 90350 77800 / 6366456458 / 8882979568  (Please call between 10 am – 6 pm ONLY) Office Address: BANGALORE CENTRE: IASbaba’s TLP Centre 2– No. 1443/1444, 2nd Floor, Above Carzspa, Ganapati Circle, Chandra Layout, Vijaynagar, Bangalore 560040. DELHI CENTRE: IASBABA, 5B, Pusa Road, Karol Bagh, New Delhi – 110005. Landmark: Just 50m from Karol Bagh Metro Station, GATE No. 8 (Next to Croma Store)    All the Best :) IASbaba

DAILY CURRENT AFFAIRS IAS | UPSC Prelims and Mains Exam – 9th October 2020

Archives (PRELIMS + MAINS FOCUS) Poverty and shared Prosperity 2020: Reversals of Fortune Part of: GS Prelims and GS-I – Social issues In news Recently, Poverty and Shared Prosperity Report was released.  Released by: The World Bank biennially According to the report, Global extreme poverty is expected to rise for the first time in 20 years because of the disruption caused by COVID-1 Key takeaways  The report provides the latest and most accurate estimates on trends in global poverty and shared prosperity. For more than two decades, extreme poverty was steadily declining.  Now, for the first time in a generation, the quest to end poverty has suffered its worst setback. The pandemic may push another 88 million to 115 million into extreme poverty or having to live on less than $1.50 per day.  Some 9.1% to 9.4% of the world will be affected by extreme poverty in 2020. Natural Gas Marketing Reforms approved  Part of: GS Prelims and GS-III – Resources In news The Union Cabinet has approved ‘Natural Gas Marketing Reforms’ to move towards gas based economy. Key takeaways  Aim of the Policy: (1)To provide standard procedure for sale of natural gas in a transparent and competitive manner; (2) To discover market price by issuing guidelines for sale by contractor through e-bidding. Advantages: (1) This will bring uniformity in the bidding process across the various contractual regimes and policies; (2) Ambiguity shall be removed; (3) It shall contribute towards ease of doing business.  Affiliate companies can also participate in the bidding process to make it more open and transparent.  The policy will also grant marketing freedom to the Field Development Plans (FDPs) of those Blocks in which Production Sharing Contracts already provide pricing freedom. Pusa Decomposer to be used on trail basis Part of: GS Prelims and GS-III – Environment  In news According to Environment Ministry, Pusa Decomposer will be used on a trial basis in Punjab, Haryana, Uttar Pradesh and Delhi for 2020. Key takeaways  The burning of paddy stubble has been a cause of concern for the past several years as it contributes to air pollution.  It is a common practice in October and November across North West India, primarily in states of Punjab, Haryana, and Uttar Pradesh.  This is done to quickly clear crop residue from their fields before planting the rabi wheat crop. Important value additions  Pusa Decomposer ‘Pusa Decomposer’ is developed to tackle the issue of stubble burning.  Developed by: Indian Agriculture Research Institute (IARI). It is a fungi-based liquid solution that can soften hard stubble so that it can be easily mixed with soil in the field to act as compost. It will also help in retaining the essential microbes and nutrients in soil that are otherwise damaged due to the burning.  Miscellaneous Nobel Prize for Literature 2020 announced  The 2020 Nobel Prize for Literature has been awarded to American poet Louise Glück. A professor at Yale University, Glück made her debut in 1968 with her collection titled 'Firstborn'.  She is seen as one of the most prominent poets and essayists in American contemporary literature. She won the Pulitzer Prize in 1993 for her collection The Wild Iris and the National Book Award for her latest collection, Faithful and Virtuous Night, in 2014. She is the fourth woman to win the Nobel Literature Prize in the past decade. (MAINS FOCUS) RIGHTS/ JUDICIARY / GOVERNANCE Topic: General Studies 2: Functions and responsibilities of various Constitutional Bodies. Ethics in Public administration:  CAG: Keeping vigil even during unusual times Context: With the nation spending substantial resources to manage the pandemic the role of the supreme audit institution of India, the Comptroller and Auditor General (CAG) of India has assumed the significance. Karnataka Case - How emergency situation is misused for corrupt practices? Emergency calls for attention on outcomes rather than rules In the expediency of saving lives and alleviating suffering, there can be reasonable exceptions to compliance with established rules and standard operating procedures. As a result, questions of inconsistencies are likely to be overlooked. In Karnataka, there was political allegation that funds (to the tune of ₹2,000 crore) were siphoned off to purchase inferior quality of PPEs, sanitisers, ventilators, masks and other equipment at prices higher than those prevailing in the market In August 2020, the Karnataka State Legislature’s Public Accounts Committee (PAC) directed the CAG to conduct a special audit into the purchase of COVID-19 equipment The panel also asked the CAG to ‘conduct an audit of expenditure incurred by the State government under the State Disaster Response Fund (SDRF). The government had used the SDRF amount for purchase of equipment in various districts’ What is the role of CAG during these times? Mandate: The statutory responsibility of CAG as an independent, objective, and non-partisan constitutional authority includes appraising disaster preparedness, ensuring that management, mitigation operations, procedures are complied with, and proper internal controls are in place. Realigning to COVID-times: The CAG has issued an order creating a new vertical — health, welfare and rural development on June 1 restructuring the office of the Director General of Audit, Central Expenditure Need to audit Health related expenditure: It is necessary that the CAG undertakes performance audits of COVID-19 related procurements, the Central Government Health Scheme (CGHS) and Employee State Insurance (ESI) hospitals.  Audit objectives may include the procurement of equipment and drugs for CGHS wellness centres and polyclinics, laboratories and hospitals.  Quality of Governance: As the CAG’s performance audits are driven by economy, efficiency and effectiveness, the audit should also focus on expense tracking and achievement of outputs and outcome, in qualitative and quantitative terms. Leveraging Technology: The entire process of procurement of COVID-19-related equipment and drugs, proper documentation, and compliance with rules and regulations can be streamlined with data analytics and AI Significance of CAG Auditing during COVID-19 times Helps Prevent Spread of Pandemic: Auditing of hospitals, dispensaries and labs is expected to provide the assured health-care services including infection control and hygiene. Disaster Management: If all the major purchases by government entities at all levels are audited by the CAG, there can be substantial improvement in disaster management.  Good Governance: CAG audit will usher in better transparency, integrity, honesty, effective service delivery and compliance with rules and procedures and governance. Long term benefits: Audit recommendations can contribute improvements in various aspects of disaster preparedness, management and mitigation. Conclusion With corruption likely in pandemic management, the CAG’s audit can ensure checks and balances in the health sector Connecting the dots: Election Commission of India’s role in conducting elections during Pandemic WOMEN/ GOVERNANCE/ SCIENCE & TECH Topic: General Studies 2,3: Government policies and interventions for development in various sectors and issues arising out of their design and implementation Mechanisms, laws, institutions and Bodies constituted for the protection and betterment of these vulnerable sections.  Science and Technology- developments and their applications and effects in everyday life. Assisted Reproductive Technology Bill – Part I Context: Union Health Minister introduced the Assisted Reproductive Technology (Regulation) Bill, 2020 (Bill) in the Lok Sabha on September 14. Aim of the Bill To regulate ART banks and clinics Allow safe and ethical practice of ARTs  Protect women and children from exploitation To supplement the Surrogacy (Regulation) Bill, 2019 (that awaits consideration by the Rajya Sabha) Some of the key Provisions of the bill are: Definition of Assistive Reproductive Technology The Bill defines ART to include all techniques that seek to obtain a pregnancy by handling the sperm or the oocyte (immature egg cell) outside the human body and transferring the gamete or the embryo into the reproductive system of a woman. Examples of ART services include gamete (sperm or oocyte) donation, in-vitro-fertilisation (fertilising an egg in the lab), and gestational surrogacy (the child is not biologically related to surrogate mother).   ART services will be provided through: (i) ART clinics, which offer ART related treatments and procedures, and (ii) ART banks, which store and supply gametes. Regulation of ART Clinics and ART Banks The Bill provides that every ART clinic and bank must be registered under the National Registry of Banks and Clinics of India.  The National Registry will be established under the Bill and will act as a central database with details of all ART clinics and banks in the country.   State governments will appoint registration authorities for facilitating the registration process.   Clinics and banks will be registered only if they adhere to certain standards (specialised manpower, physical infrastructure, and diagnostic facilities).   The registration will be valid for five years and can be renewed for a further five years.  Conditions for gamete donation and supply A bank can obtain semen from males between 21 and 55 years of age, and oocytes from females between 23 and 35 years of age.   An oocyte donor should be an ever-married woman having at least one alive child of her own (minimum three years of age).   The woman can donate oocyte only once in her life and not more than seven oocytes can be retrieved from her.   A bank cannot supply gamete of a single donor to more than one commissioning couple (couple seeking services). Conditions for offering ART services ART procedures can only be carried out with the written informed consent of both the party seeking ART services as well as the donor.   The party seeking ART services will be required to provide insurance coverage in the favour of the oocyte donor (for any loss, damage, or death of the donor).   A clinic is prohibited from offering to provide a child of pre-determined sex.   The Bill also requires checking for genetic diseases before the embryo implantation. Rights of a child born through ART: A child born through ART will be deemed to be a biological child of the commissioning couple and will be entitled to the rights and privileges available to a natural child of the commissioning couple.   A donor will not have any parental rights over the child. National and State Boards:  The Bill provides that the National and State Boards for Surrogacy constituted under the Surrogacy (Regulation) Bill, 2019 will act as the National and State Board respectively for the regulation of ART services.  Key powers and functions of the National Board include: Advising the central government on ART related policy matters Reviewing and monitoring the implementation of the Bill Formulating code of conduct and standards for ART clinics and banks Overseeing various bodies to be constituted under the Bill.   Analysis of the provisions of the bill will be taken up in Part II of this article (TEST YOUR KNOWLEDGE) Model questions: (You can now post your answers in comment section) Note:  Correct answers of today’s questions will be provided in next day’s DNA section. Kindly refer to it and update your answers.  Comments Up-voted by IASbaba are also the “correct answers”. Q.1 Consider the following statements regarding the Conventional source of energy: The conventional sources of energy are the fossil fuels which took millions of years for the formation of fossil fuels; hence they are limited and non-renewable. Fossils are remains of organisms that lived in the past and fossil fuels are plants that got buried under earth that became rock over years. Which of the following statement(s) is/are correct? 1 Only 2 Only Both 1 and 2 Neither 1 nor 2 Q.2 Poverty and Shared Prosperity Report was recently launched by which of the following organisation? World bank International Monetary Fund World Economic Forum None of the above Q.3 Pusa Decomposer is associated with which of the following? Thermal Power Plants Organic Farming Stubble Burning Automobile Exhaust ANSWERS FOR 8th October 2020 TEST YOUR KNOWLEDGE (TYK) 1 B 2 B 3 C 4 D Must Read About Nobel Prize in Chemistry: The Hindu About RBI’s Ni-Monthly Monetary Policy review: The Hindu About SC recent judgement on right to protest: The Hindu

Daily Prelims CA Quiz

UPSC Quiz - 2020 : IASbaba's Daily Current Affairs Quiz 9th Oct 2020

For Previous Daily Quiz (ARCHIVES) - CLICK HERE The Current Affairs questions are based on sources like ‘The Hindu’, ‘Indian Express’ and ‘PIB’, which are very important sources for UPSC Prelims Exam. The questions are focused on both the concepts and facts. The topics covered here are generally different from what is being covered under ‘Daily Current Affairs/Daily News Analysis (DNA) and Daily Static Quiz’ to avoid duplication. The questions would be published from Monday to Saturday before 2 PM. One should not spend more than 10 minutes on this initiative. This is a part of our recently launched, NEW INITIATIVE IASbaba’s INTEGRATED REVISION PLAN (IRP) 2020 – Road Map for the next 100 Days! FREE INITIATIVE! We will make sure, in the next 4 months not a single day is wasted. All your energies are channelized in the right direction. Trust us! This will make a huge difference in your results this time, provided that you follow this plan sincerely every day without fail. Gear up and Make the Best Use of this initiative. Do remember that, “the difference between Ordinary and EXTRA-Ordinary is PRACTICE!!” To Know More about the Initiative -> CLICK HERE SCHEDULE/DETAILED PLAN – > CLICK HERE Important Note: Don't forget to post your marks in the comment section. Also, let us know if you enjoyed today's test :) After completing the 5 questions, click on 'View Questions' to check your score, time taken and solutions. To take the Test - Click Here

Important Articles

[MOST TRUSTED] IASbaba’s TLP Plus (+) MAINS Mentorship-Based Answer Writing Program for UPSC Mains 2020 – (Both OFFLINE & ONLINE) - ADMISSIONS OPEN!!

Dear Friends, How do batsmen of international repute like Virat Kohli or Steve Smith are able to pierce the gaps with their cracking cover drives? How to fierce pacers like Bumrah or Mitchell Starc can bowl bullet like yorkers on the trot? Well, the answer is simple- they have the skills to do what they do and they have perfected their skills through years of practice. Although a cliche, you need to understand that practice does make a man (or woman) perfect. When you listen to commentators praising Virat Kohli for his abilities and that he can play his shots while sleeping is a testimony to the significance of practice. While you might be a good cricketer, you are supposed to practice something different to succeed in the Mains examination. Yes, we are talking about answer writing. People tend to forget a simple fact - what you have read and understood doesn’t matter if you don’t have the ability to express your learnings in words. The only tool that you have to impress the examiner is the answer booklet where you write the answers. The examiner, while evaluating your answers, won’t know your gender, age, background, personality, sense of humour and of course the hard work that you might have put in during the preparation process. He will judge you only through the prism of your answers. While it may appear unfair, you need to quickly adapt to this reality and buckle up for answer writing practice, a lot of it. It is not going to be easy by the way. Most of us hate to write. In the age of social media and cellphones, we are quicker and more efficient in typing on qwerty keys than writing with a pen. The initial days will produce a lot of friction. You will tend to loose interest midway, thinking that you will manage to write like Amartya Sen or Shashi Tharoor during Mains. Let us tell you that it doesn’t work that way. Reading and writing are two different ballgames. People may be good in both or either of the two. Depending upon how comfortable you are with your writing and reading skills, you need to prioritise your time. So, in order to give direction to your mains preparation, we are starting the most trusted Mains program for answer writing. Yes, we are talking about Think Learn and Perform Plus (TLP+) Program for UPSC Mains 2020! Here, we bring you the most trusted and awaited program of IASbaba – Think Learn and Perform (TLP)+ for UPSC Mains 2020. For the past 5 years, TLP has become synonymous with quality and effectiveness. The testimony and recommendation of toppers and a hit ratio of more than 82% in Mains (in the last 4 Years)  speak volumes about the quality of questions that IASbaba frames. In TLP, you will get a mix of thought provoking, opinion oriented, highly analytical and most probable questions apart from few direct questions to prepare you for any scenario that UPSC can throw at you. You can see it yourself here: 2016, 2017, 2018, 2019 Topper's Testimonials Previous Next In UPSC 2019, 150+ (from ILP and TLP ONLY) students from IASbaba got an Interview call. It’s a Proud moment!  All Credit goes to the efforts of our students put in the right direction!  100+ Ranks UPSC CSE 2019 Topper’s from IASbaba – 5 in Top 20 Ranks makes it even more Special! -> CLICK HERE Now, coming to the – 1. FEATURES OF TLP (+) PLUS: This is one of the most intensive and comprehensive Answer Writing Program of IASbaba designed with some unique features exclusively  for UPSC Mains 2020 keeping the paucity of time between Prelims and Mains in mind. The Program will start from 12th October, 2020 (Monday). This programme is available in both offline and online modes. Please Note: Both TLP+ & TLP Full Length Mock (FLM) Tests are available OFFLINE & ONLINE. But given the present conditions you can take the programme ONLINE and when conditions are back to normal you can change to OFFLINE Mode. Mentors will be available in the offline centres.  TLP+ 2020 is a MAINS Mentorship-Based Answer Writing Program for UPSC Mains 2020. One-to-one personal guidance and feedback from experts. Each evaluated copy is discussed in detail by the mentor, who will help you in identifying your strengths and weakness and give you the right direction. There are 25 Full-Length Tests (12 Full-Length Sectional Tests and 8 Full-Length Mock Tests and 5 Essay Tests). Special emphasis is given on Ethics and Essay. All the Tests are for 250 Marks, 3 hours Duration and these tests are flexible! Quality of Questions -A mix of static and analytical questions which will be on the lines of UPSC pattern. The tests are designed to help you in developing the thought process and on the spot thinking which is very important while answering opinion based questions in the actual exam scenario. Most of you would vouch for the quality of TLP questions and this year’s results stands testimony to it. Detailed Synopsis - How to Approach the questions based on the demand of the question will be mentioned in the Synopsis. There will be a special emphasis on approach to analytical questions and alternative introductions one can come up with, to the same question. All the important pointers will be given along with value addition. Evaluation and Ranking to be done and list to be published for both Online and Offline Combined. This time, we have synced the schedules of our TLP (Free Initiative) with TLP+ Programme (paid) so that, those who will be enrolling for TLP+ can also use the free initiative to daily hone their answer writing skills and at the same time give full –length tests every week, making it a comprehensive answer writing program for UPSC Mains 2020! Please Note, On Every Saturday, there will be GS tests and on every 4th Saturday along with Ethics (GS-4) will be Essay test. Only the 1st Test will be starting on 10th October, the remaining tests will be held on every Saturday’s. If Answer Copies are submitted on the same day or within 24 hours of the exam, it will be evaluated and given before the next test. For others, copies will be given within maximum of 7 Days. A Separate platform has been designed for ONLINE students. Tests will be uploaded on this Platform on the day of the exam and you will be given a time-domain to write and upload your answer sheets. Evaluated answer sheets will be uploaded back on your profile before the next scheduled test, provided you are submitting the Answer copies within 24 hours. This will help us in timely evaluation and publication of Offline + Online Ranking. **SPECIAL FEATURES** Apart from the USP of all IASbaba’s Programs – 'Mentorship', where IASbaba is the pioneer of this model, the following are the special/unique features added to TLP+ this year. Weekly Answer-Writing Enrichment Sessions with Mohan Sir (IASbaba Founder). These sessions will immensely help you as Mohan sir will share his years of experience on the thought process, Answer Writing skills and what it takes to top the mains exam, which he has done consistently over the years. These Sessions will start from the 3rd Week of the Program. Students can get directly get in touch with Mohan Sir apart from the regular mentorship provided by your respective mentor. Weekly Value added notes (VAN’s) – on Contemporary and Current Affairs Topics which will act as Mains fodder points and can be used directly in the UPSC Mains Examination. A few highly specialised sessions under the SERIES OF INTERACTIVE LECTURES (SOIL) to help you grow your analytical ability in areas like Ethics, Essay, Answer-Writing Strategy (GS1, GS2, GS3), International Relations and among others will be taken by Mohan Sir, People in Services and Toppers on Pro-bono basis. For Sample [ESSAY and GS ANSWER COPIES] of SARANYA RAMCHANDRAN Rank 36 UPSC CSE 2019 TOPPER- IASbaba’s TLP Connect Student -> CLICK HERE 2. EXCLUSIVE MODULE : TLP+ Full-length Mocks (FLM) For those who are interested in taking ONLY Full-length Mock tests along with mentorship, we have an Exclusive Module, TLP+ Full-length Mocks. It will consist of 10 Full-length Mock Tests comprising of Essay, GS1, GS2, GS3, GS4 on the similar lines of UPSC Mains Exam. Mentorship: One-to-one personal guidance and feedback from experts. Each evaluated copy is discussed in detail by the mentor, who will help you in identifying your strengths and weakness and give you the right direction. The First Full-Length Mock will be held in 1st Week of November and the Second Full-Length Mock will be held in 1st Week of December. These Tests are Flexible! Schedule DOWNLOAD THE SCHEDULE of TLP+ 2020 Mains Mentorship Programme DOWNLOAD THE SCHEDULE of Exclusive Model TLP+ 2020 Full Length Mocks (FLM) We invest our hearts and soul in this initiative. We have come a long way in the past few years. We try our best to make the questions as authentic and relevant as possible. In return, we want your best effort. We have seen consistency in paying off sincere candidates in the past. We believe in you and we are sure that for all the hard work that we put into this initiative, you will reciprocate by being regular and determined. So here, we would like to reiterate the tagline of IASbaba’s TLP Program – "Write to Write Your Future ..!!" Success at anything will always come down to this - Focus & Effort, and we control both.. !! Fees Details TLP PLUS Rs. 21,000 + 18% (GST) = 24,780/- Make the Payment for ONLINE Programme Make the Payment for OFFLINE Programme TLP PLUS (FLM) Rs. 8,000 + 18% (GST) = 9,440/- Make the Payment for ONLINE Programme Make the Payment for OFFLINE Programme For TLP Plus (ONLINE), once payment is done, an acknowledgment will be sent via mail. The Mains Test Platform will be active 2 days before the Test (i.e., 10th October), you will also receive Login credentials on the same day. FOR ANY QUERIES (Related to TLP+): You can reach us on Email id: tlp@iasbaba.com Mobile No: 63664 56458 / 88829 79568 / 98992 91288 (Please call between 10 am – 6 pm ONLY) Office Address: BANGALORE CENTRE: IASbaba’s TLP Centre – No. 1443/1444, 2nd Floor,  Ganapati Circle, Chandra Layout, Vijaynagar, Bangalore 560040. DELHI CENTRE: IASBABA, 5B, Pusa Road, Karol Bagh, New Delhi – 110005. Landmark: Just 50m from Karol Bagh Metro Station, GATE No. 8 (Next to Croma Store) All the Best :) IASbaba

Important Articles

LAST DAY to REGISTER for ‘Super 50’ SCHOLARSHIP TEST for IASbaba’s TLP CONNECT 2021 and TLP PLUS(+) 2021 Mentorship Program (OFFLINE and ONLINE)

Hello Everyone, Firstly, thank you for an amazing response on our TLP Connect (Prelims + Mains+ Interview) Test Series Based INTEGRATED Mentorship Based Programme and TLP+ 2021 Mains Mentorship Program. TLP Connect and TLP+ 2021 Programs are meant for aspirants preparing for UPSC 2021. To Know more details about these Programs -> CLICK HERE We are just 2 days away from the Super 50 Scholarship Test. This post is a reminder to all the students, that the Scholarship Test for ‘Super 50’- TLP Connect 2021 and TLP+ 2021 is going to be conducted on 10th October 2020. Last day to register for Super 50 is tomorrow 9th October 2020 (by 8PM) [wpcdt-countdown id="79988"] Since the number of registrations has shot up quickly for Super 50, we have decided to have 2 Slots for taking the test, so that the exam goes smooth without any hiccups. 1. So, what is Super 50? Super 50 is a Scholarship Test. Based on the performance in the Test, the top 25 students will be given 100% fee waiver and the next 25 students will be given 50% discount. 2. What is the Syllabus for Super 50? It is a DESCRIPTIVE TEST. The test will be conducted on 10th October 2020. An ESSAY and ETHICS TEST will be conducted on 10th October 2020. The test will consist of 2 Case Studies (Ethics)- 40 Marks and an Essay – 120 Marks. Total 160 Marks. 3. Timings & Duration of the Test? Super 50 will be conducted on 10th October 2020. It will be conducted in 2 Slots. 1st Slot: Timings: 11 AM - 12.30 PM Duration of the Test: 1 hour 30 minutes. 2nd Slot: Timings: 6 PM - 7.30 PM Duration of the Test: 1 hour 30 minutes. IMPORTANT NOTE: 1. Online Link to take up the Scholarship Test -> CLICK HERE Please note that - the above test link will be activated or you will be able to login to the test platform ONLY 1 hour before the Test (i.e., at 10 Am on 10th October). 2. ONLY handwritten scanned answer copies will be considered for evaluation. There will be no provision for uploading the typed answer-copies. 4. How to Register for SUPER 50? All the students will be eligible for the test. No pre-conditions. FREE Registration !!  TO REGISTER FOR ‘SUPER 50’ -> CLICK HERE The last date to register for SUPER 50 is 9th October 2020 (by 8PM) 5. Announcement of the results? The result will be communicated via Email/Phone Call on 13th October (Tuesday by 8 pm)to all the selected candidates. To Know More about TLP Connect/Plus (2021) Test-Series (Pre+Mains) Based Mentorship Program -> CLICK HERE To Know More about IASbaba's 2021 Courses -> CLICK HERE For ONLINE - TLP Connect/TLP+ 2021 PAYMENT -> CLICK HERE For OFFLINE - TLP Connect/TLP+ 2021 PAYMENT -> CLICK HERE   FOR ANY QUERIES (Related to TLP): You can reach us on Email id: support@iasbaba.com or tlp@iasbaba.com Mobile No: 63664 56458 / 88829 79568 (Please call between 10 am – 6 pm ONLY) Office Address: BANGALORE CENTRE: IASbaba’s TLP Centre – No. 1443/1444, 2nd Floor,  Ganapati Circle, Chandra Layout, Vijaynagar, Bangalore 560040. DELHI CENTRE: IASBABA, 5B, Pusa Road, Karol Bagh, New Delhi – 110005. Landmark: Just 50m from Karol Bagh Metro Station, GATE No. 8 (Next to Croma Store) All the Best :) IASbaba

Important Articles

[UPSC Mains 2020 - Daily Mains Answer Writing Programme] IASbaba’s Think, Learn and Perform (TLP) 2020, Phase II (ONLINE FREE Initiative) - Most-Trusted Program for MAINS 2020- Starting from 12th Octo

Dear Friends, Finally, the most-awaited and trusted programme of IASbaba for UPSC Mains 2020 is here!   For a surgeon, it takes years of practice and experience to master the art of surgery. The precision required to perform complex surgeries comes from the confidence acquired in the years gone by. The same holds true for other professions and areas of expertise. What we do at IASbaba doesn’t only require precision but also anticipation. After all, we deal with an examination conducted by UPSC, a body known for its unpredictability. Continuing the legacy of an initiative that has become synonymous with terms like accurate, relevant and effective, IASbaba is pleased the announce the launch of its Flagship ONLINE (FREE) INITIATIVE - Think, Learn and Perform (TLP) - Phase II for Mains 2020. Mains Answer Writing is a very important component of your preparation and the last 5 years have shown the mettle of IASbaba Mains Answer Writing Programme - TLP in creating an ecosystem of knowledge that can be as close as it gets to the demands of UPSC. The testimony and recommendation of toppers and a hit ratio of more than 82% in Mains (in the last 4 Years) speak volumes about the quality of questions that IASbaba frames You can see it yourself here: 2016, 2017, 2018, 2019 Download the Compilations of TLP 2020 (Phase I) Compilations - CLICK HERE Topper’s Testimonial Previous Next Philosophy of TLP – Mains Daily Answer Writing Program (A FREE Initiative) The driving force of TLP is the most imperative skill required in Mains- the ability to express your knowledge effectively and convincingly in 150 to 250 words in the stipulated 180 minutes that you get in a Mains paper. It is not as easy as it sounds even for those who have fair command over the Mains syllabus. In fact, without adequate answer writing practice, one finds herself at sea during Mains. It becomes difficult to choose the right information and even more difficult to get the right words to express that information. This is where TLP comes handy. It gives you 75 days of daily rigorous practice, enough to score well in Mains. Through the course of TLP, you get exposed to more than 350 high-quality questions, receive expert feedback and peer reviews and get elaborate synopsis to complement and expand your understanding. This way, TLP provides an end to end solution to your answer writing needs. How does the TLP Program work? We, at IASbaba, are the pioneers of the art of taking out the best from students. We have been doing it for the past 6 years. The idea is simple - we post questions, you write and upload your answers. Your answers get reviewed by IASbaba or your peers on the answer writing platform, we provide detailed and well-researched synopsis and you get access to hundreds of answers written by your fellow aspirants. You learn in this process. You improve your speed and language. You start contributing in the discussions and within a few weeks time, you find yourself much more comfortable, so much so, that daily answer writing becomes an ‘addiction’. The quality of questions that we post is unmatched. That our answer writing initiative gets recommended by toppers should be enough to make you relax with respect to the quality of this initiative. IMPORTANT HIGHLIGHTS OF TLP ( Daily Mains Answer Writing Program – UPSC 2020) The Program will start from 12th October 2020 (Monday). It will run for 6 days a week. Sunday is a holiday – you can use this time for Revision and Optional. Practical Planning– The Daily Schedule comes with Daily Targets with adequate time for your Optional Preparation. On Daily Basis – 5 Mains Questions will be posted covering GS1, GS2, GS3. Out of 5 Questions, 3 will be from Static and 2 from Current Affairs. Special Emphasis on GS4 (Ethics) – Every week (on Friday’s) 5 Questions will be posted covering Ethics Theory. Once you gain confidence and strengthen your foundation we will be posting Case Studies in the later half of the program. Special Emphasis on Essay – On Every Saturday’s there will be Essay topics posted.  Philosophical and Theme-Based Essay’s (Polity, Governance, Economy, International Relations, Art & Culture, Sports, Science & Technology, Environment etc.) alternating every week. Your answers get Reviewed by IASbaba or your peers on the answer writing platform Detailed and well-researched Synopsis/Model Answers will be published on the same day by 9 PM on daily basis. Best Answers will be featured. You will get access to hundreds of answers written by your fellow aspirants on daily basis. PLEASE NOTE: Those who show Consistency in Answer writing, actively participate in the program, and get their answers featured on regular basis will get a call (guidance) from the core team of IASbaba. DOWNLOAD THE SCHEDULE -> CLICK HERE We invest our hearts and soul in this initiative. We have come a long way in the past few years. We must have posted more than 3000 questions and synopsis since the inception of the answer writing initiative. We hardly repeat our questions and try our best to make the questions as authentic and relevant as possible. In return, we want your best effort. We have seen consistency in paying off sincere candidates in the past. We believe in you and we are sure that for all the hard work that we put into this initiative, you will reciprocate by being regular and determined. So, before you embark upon this journey, let us give you a pearl of wisdom on answer writing: “Start writing, no matter what. The water does not flow until the faucet is turned on.” All the Best :) IASbaba

DAILY CURRENT AFFAIRS IAS | UPSC Prelims and Mains Exam – 8th October 2020

Archives (PRELIMS + MAINS FOCUS) Production Linked Incentive Scheme (PLI) For Large Scale Electronics Manufacturing Part of: GS Prelims and GS-III – Technology In news 16 eligible applicants under the PLI Scheme were approved recently. Ministry: Ministry of Electronics and Information and Technology (MeitY)  Important value additions Production Linked Incentive Scheme (PLI) for Large Scale Electronics Manufacturing  It was notified on 1st April, 2020. It extends an incentive of 4% to 6% on incremental sales of goods under target segments that are manufactured in India to eligible companies, for a period of five years (base year- FY2019-20. Over the next 5 years, the approved companies under the PLI Scheme are expected to lead to total production of more than INR 10.5 lakh crore. The companies approved under the scheme are expected to promote exports significantly. The companies approved under the scheme will bring additional investment in electronics manufacturing to the tune of INR 11,000 crore. The National Clinical Management Protocol released Part of: GS Prelims and GS-II – Medicine In news The National Clinical Management Protocol based on Ayurveda and Yoga for the management of Covid-19 was released. Ministry:  Jointly by Ministry for Health and Ministry for AYUSH. Prepared by: Experts and other national research organizations According to: Report & Recommendations of the Interdisciplinary Committee Key takeaways The protocol contains the details self-care guidelines for preventive health measures to help protect against COVID-19 pandemic. Clinical studies have established some Ayurveda interventions like Ashwagandha, Laung & Giloy as anti-inflammatory, antiviral & immunity-modulating which help protect against COVID-19. Measures: Drinking herb-infused warm water or turmeric milk, steam inhalation, gargling, moderate exercise and a set of medicines to be used for prophylaxis and treatment. World Cotton Day observed Part of: GS Prelims and GS-I – Agricultural crops In news The 2nd World Cotton Day was observed on 7th October, 2020. Key takeaways World Cotton Day was launched at the initiative of the Cotton-4 (Benin, Burkina Faso, Chad and Mali) by the World Trade hosted on 7th October 2019. It was launched in collaboration with the secretariats of the UN, FAO, UNCTAD, the International Trade Centre (ITC) and the International Cotton Advisory Committee (ICAC). Objective: (1) To recognize the importance of cotton as a global commodity grown in over 75 countries across 5 continents; (2) To highlight its central role in job creation and maintaining economic stability in several least-developed countries. Important value additions Cotton Four Cotton is discussed at the WTO for: (1) the trade reforms needed to address subsidies and high trade barriers for cotton; (2) the assistance provided to the cotton sector in developing countries. These various tracks of discussion have been developed over the years as a response to a series of proposals to address the cotton sector tabled by four African countries — Benin, Burkina Faso, Chad and Mali — known as the Cotton Four or C4. Ratification of seven chemicals under Stockholm Convention on Persistent Organic Pollutants (POPs)  Part of: GS Prelims and GS-III – Pollution In news The Union Cabinet has approved the ratification of seven chemicals listed under Stockholm Convention on Persistent Organic Pollutants (POPs). Key takeaways These are: (1) Chlordecone; (2) Hexabromobiphenyl; (3) Hexabromodiphenyl ether and Heptabromodiphenylether; (4) Tetrabromodiphenyl ether and Pentabromodiphenyl ether, (5) Pentachlorobenzene; (6) Hexabromocyclododecane; (7) Hexachlorobutadiene. The ratification process would enable India to access Global Environment Facility (GEF) financial resources in updating the National Implementation Plan (NIP). Important value additions Stockholm Convention It is a global treaty to protect human health and environment from POPs.  POPs are chemical substances that persist in the environment, bio-accumulate in living organisms and have the property of long-range environmental transport. Exposure to POPs can lead to cancer, damage to nervous systems, diseases of immune system, reproductive disorders etc. India had ratified the Stockholm Convention in 2006.  The Ministry of Environment had notified the 'Regulation of Persistent Organic Pollutants Rules in 2018 under the provisions of Environment (Protection) Act, 1986. Examples of persistent organic pollutants include: (1) Aldrin; (2) Chlordane; (3) DDT; (4) Dieldrin; (5) Endrin; (6) Heptachlor; (7) Hexachlorobenzene; (8) Mirex SC gives judgement regarding Right to Protest Part of: GS Prelims and GS-II – Fundamental Rights; Judiciary In news The Supreme Court has recently highlighted that occupying public places for protests is not acceptable and such a space cannot be occupied indefinitely. Key highlights of the judgement The verdict came after a plea was filed against the Anti-Citizenship Amendment Act protests which had led to blocking of a road in Shaheen Bagh in Delhi last December. The judgment upheld the right to peaceful protest against a law  It also made clear that public ways and public spaces cannot be occupied, especially indefinitely. The rights to free speech and peaceful protest were “treasures” but also subject to reasonable restrictions imposed in the interests of sovereignty, integrity and public order.  The right of the protester has to be balanced with the right of the commuter. They have to co-exist in mutual respect. The bench also said that it was entirely the responsibility of the administration to prevent encroachments in public spaces. Nobel Prize in Chemistry 2020 announced Part of: GS Prelims and GS-III – Science and Technology In news Emmanuelle Charpentier of France and Jennifer Doudna of the U.S. won the Nobel Chemistry Prize for the gene-editing technique known as the CRISPR-Cas9 DNA snipping “scissors” tool. It is the first time a Nobel science prize has gone to a women-only team. Key takeaways Using the CRISPR-Cas9 DNA snipping “scissors” tool, researchers can change the DNA of animals, plants and microorganisms with extremely high precision. The CRISPR-Cas9 tool has already contributed to significant gains in crop resilience, altering their genetic code to better withstand drought and pests. The technology has also led to innovative cancer treatments. Many experts hope it could one day make inherited diseases curable through gene manipulation. CRISPR’s relative simplicity and widespread applicability has, however, triggered the imaginations of rogue practitioners.  In 2018 in China, scientist He Jiankui caused an international scandal when he used CRISPR to create what he called the first gene-edited humans. Kasturi Cotton: India’s premium Cotton Part of: GS Prelims and GS-I – Agricultural Crops In news The 1st ever Brand & Logo for Indian Cotton was launched on 2nd World Cotton Day. Launched by: Ministry of Textiles Now India’s premium Cotton would be known as ‘Kasturi Cotton’ in the world cotton Trade. The Kasturi Cotton brand will represent Whiteness, Brightness, Softness, Purity, Luster, Uniqueness and Indianness. Important value additions Cotton Cotton is one of the principal commercial crops of India.  It provides livelihood to about 6 million cotton farmers. India is the 2nd largest cotton producer. It is the largest consumer of cotton in the world. A mobile app, “Cott-Ally” has been developed by Cotton Corporation of India (CCI) for providing latest news regarding weather condition, Crop situation and best farm practices. (MAINS FOCUS) RIGHTS/ JUDICIARY / GOVERNANCE Topic: General Studies 2: Social empowerment and Fundamental Rights Judiciary and their role On public protests Context: The Supreme Court has found the indefinite “occupation” of a public road by the Shaheen Bagh protestors unacceptable. Brief Background of the issue The Shaheen Bagh (New Delhi) protest was a sit-in peaceful protest and an iconic dissent mounted by mothers, children and senior citizens against the passage of the Citizenship (Amendment) Act (CAA) Mainly consisting of Muslim women, the protesters at Shaheen Bagh, since 14 December 2019, blocked major road in New Delhi using non-violent resistance It became the longest protest against CAA-NRC-NPR. However, the protest became inconvenient to commuters. Petitions were filed in High Court and Supreme Court to stop the blockade caused by the protests and to shift the site of protest. Later, they were removed by the police from the site on March 24th on the wake of lockdown imposed on Pandemic What were the judgements of Supreme Court? Judgment upheld the right to peaceful protest against a law but made it unequivocally clear that public ways and public spaces cannot be occupied, and that too indefinitely.  Protests cannot be held at any places: The demonstrations expressing dissent have to be in designated places alone. Dissent in Self-Ruled Democracy: The seeds of protest and dissent were sown deep during the Freedom struggle. But dissent against the colonial rule cannot be equated with dissent in a self-ruled democracy. Reasonable restrictions: In a democracy, the rights of free speech and peaceful protest were indeed “treasured”. But these rights were also subject to reasonable restrictions imposed in the interest of sovereignty, integrity and public order. Police regulations also weighed in. Right of commuter: Fundamental rights do not live in isolation. The right of the protester has to be balanced with the right of the commuter. They have to co-exist in mutual respect Intervention by High Court was needed: The judgment said the Delhi High Court should have intervened positively and not left the situation fluid. The administration too should have talked to the protesters. Responsibility of the administration: The court held it was entirely the responsibility of the administration to prevent encroachments in public spaces. They should do so without waiting for courts to pass suitable orders. Significance of Digital Media: SC noted that Shaheen Bagh seemed typical of the many digitally-fuelled “leaderless” dissent seen in modern times. Technology and social media could both empower and weaken mass movements. Conclusion Indian democracy is best served when citizens freely express their views, mobilise and protest, but do so without undermining the rights of fellow-citizens. This will help keep the trust between differing constituencies and enhance the legitimacy of dissent Connecting the dots: Children’s Right to Protest and Safeguards for Child Witness: Click here Procedural reforms needed in Judiciary: Click here ECONOMY/ GOVERNANCE/ RIGHTS Topic: General Studies 2,3: Fundamental Rights Government policies and interventions for development in various sectors and issues arising out of their design and implementation Gig Workers and its skewed terms Context: The new Code on Social Security allows a platform worker to be defined by their vulnerability — not their labour, nor the vulnerabilities of platform work. What is Gig Economy? A gig economy is a free market system in which temporary positions are common and organizations contract with independent workers for short-term engagements Examples of gig employees in the workforce could include freelancers, independent contractors, project-based workers and temporary or part-time hires. Do You Know? Global Gig Economy Index report has ranked India among the top 10 countries. The report says there has been an increase in freelancers in India from 11% in 2018 to 52% in 2019, thanks to various initiatives including Startup India and Skill India. Issues of Gig Workers – Example of Swiggy (Food Delivery platform) Swiggy workers have been essential during the pandemic. They have faced a continuous dip in pay, where base pay was reduced from ₹35 to ₹10 per delivery order, despite braving against the odds of delivering during Pandemic Stable terms of earning have been a key demand of delivery-persons  Does new version of labour code offer any relief to Gig workers? The three new labour codes passed by Parliament recently acknowledge platform and gig workers as new occupational categories in the making Defining gig workers is done in a bid to keep India’s young workforce secure as it embraces ‘new kinds of work’, like delivery, in the digital economy. In the Code on Social Security, 2020, platform workers are now eligible for benefits like maternity benefits, life and disability cover, old age protection, provident fund, employment injury benefits, and so on. Issues with new labour codes for gig workers Platform delivery people can claim benefits, but not labour rights.  This distinction makes them beneficiaries of State programmes but does not allow them to go to court to demand better and stable pay, or regulate the algorithms that assign the tasks.  This also means that the government or courts cannot pull up platform companies for their choice of pay, or how long they ask people to work. The laws do not see them as future industrial workers. They are now eligible for government benefits but eligibility does not mean that the benefits are guaranteed. Actualising these benefits will depend on the political will at the Central and State government-levels. The language in the Code is open enough to imply that platform companies can be called upon tso contribute either solely or with the government to some of these schemes. But it does not force the companies to contribute towards benefits or be responsible for workplace issues. Conclusion There are no guarantees for better and more stable days for platform workers, even though they are meant to be ‘the future of work’. Connecting the Dots: Impact of AI on jobs Skill India program (TEST YOUR KNOWLEDGE) Model questions: (You can now post your answers in comment section) Note:  Correct answers of today’s questions will be provided in next day’s DNA section. Kindly refer to it and update your answers.  Comments Up-voted by IASbaba are also the “correct answers”. Q.1 Consider the following useful applications of CRISPR-Cas9 tool: Crop resilience Treatment of AIDS Treatment of cancer Gene-editing of humans Which of the above are true? 1 and 2 only 1 and 3 only 3 and 4 only 2 and 4 only Q.2 Kasturi Cotton was recently launched by which of the following Ministry? Ministry of Agriculture Ministry of Textiles Ministry of Micro, Small and Medium Enterprises Ministry of Consumer Affairs Q.3 Which of the following is not a Fundamental Right? Freedom of Speech and expression Right to Equality Freedom of assembly with arms Right to Pollution-free water and air Q.4 Which of the following is not a persistent organic pollutants? Heptachlor Hexachlorobenzene DDT Mercury ANSWERS FOR 7th October 2020 TEST YOUR KNOWLEDGE (TYK) 1 C 2 A Must Read About flood forecasting technology: The Hindu About Nobel Prize in Physics: The Hindu About Schools being reopened amidst Pandemic: The Hindu

TLP Mains 2020

SYNOPSIS GS Paper 4 FULL MOCK[8th October,2020] : IASbaba’s TLP (Phase 2): UPSC Mains Answer Writing

SYNOPSIS GS Paper 4 FULL MOCK[8th October,2020] : IASbaba’s TLP (Phase 2): UPSC Mains Answer Writing   1.a) Describe ‘Kant’s Deontological Theory’. (150 words) (10) Demand of the question: It expects candidates to write details of Deontological Theory as propounded by Kant.  Introduction: Immanuel Kant, the foremost philosopher of Deontology, proposed a moral law called “categorical imperative” stating that morality is derived from rationality. According to Kant there are “categorical imperatives” which are in nature of absolute commands and need to be obeyed without exception for action to be judged as ethical. Body: According to Kant, ethics based on the consequences is  based up on hypothetical imperative and do not have moral sanction. The lack of absoluteness in consequential approach makes them a matter of desire.  For instance, a Public Servant has to take a decision where among stakeholders, one’s gain is others loss e.g. In a situation of land acquisition for setting up factory, farmers livelihood is lost but at the same time there will be job creation for Youths.  In such a situation Consequential approach becomes a matter of preference for Public Servant with no objective guide to arrive at moral action. This framework has the advantage of creating a system of rules which is consistent with  expectations of  people. Deontological ethics holds that at least some acts are morally obligatory regardless of their consequences for human welfare. This framework has the advantage of creating a system of rules that has consistent expectations of all people. If an action is ethically required, it would apply to every person in a given situation. Thus, speaking truth in all situations is categorical imperative which is applicable universally. This approach is helpful in resolving dilemmas a civil servant may face during performance of duty where a course of action may resolve a genuine problem by going against established procedure. The categorical imperative of giving precedence to duty helps in resolving such dilemmas. By focusing on a person’s intentions, it also places ethics entirely within our control – we can’t always control or predict the outcomes of our actions, but we are in complete control of our intentions. The advantages of 'Deontological approach' are innumerable. However, due to its theoretical perspective it has some of the drawbacks' as mentioned below: Notion of finding a universal moral standard has been criticised by some of the philosophers who argue that, because of cultural differences in societies, arriving at absolute standards of morality is hard to achieve. Bioethical decisions in areas such as abortion, euthanasia, cloning, organ harvesting, end-of-life decisions, etc. are against the ethics of a medical practitioner, yet practiced for the greater good. It poses a challenge to utilitarianism as it ignores what is at stake in terms of consequences. Kant, for example, argued it would be unethical to lie about the location of our friend, even to a person trying to murder them! This approach may require actions which are known to produce harms, even though they are strictly in keeping with a particular moral rule. For example, in situations like Second World War, where German bureaucrats may justify their actions as result of duty or obligations cast up on them. Conclusion: Judging a person's behaviour on the basis of a single absolute imperative is likely to produce undesirable situation. Hence, the rigidity of Deontology can be tackled by using threshold deontology where the decision is based on the nature of situation. 1. (b) Define the following virtues and describe their significance in civil services:(200 words) (10) (i)Goodwill (ii)Altruism   (iii)Dispassion  (iv)Objectivity    Demand of the question: It expects candidates to define and state the significance of the aforesaid virtues in civil services.  (i)Goodwill Definition: A benevolent interest or concern for others is known as goodwill. Goodwill is an important virtue to affirm the dignity and respect of individual. Significance: Active participation of citizens is essential for the effective implementation of public policy on ground. Goodwill ensures the civil servants are true to the service of citizens and this virtue when recognized by citizens can help persuade them in bringing changes by acting on the sheer goodwill of the bureaucrat. Example: Kerala’s Kannur became India’s first plastic-free district, that too in just five months through efforts of District Collector, Mir Mohammed Ali who initiated a focused campaign towards phasing out plastic carry-bags and disposables. (ii)Altruism   Definition: Altruism or selflessness is the principle or practice of concern for the welfare of others. It is a traditional virtue in many cultures and a core aspect of various religious traditions and secular worldviews. Significance: Altruism has deep roots in human nature because helping and cooperation promote the survival of our species. Altruism ensures a civil servant works for the welfare of the people and it is the core value that derives one towards effectively performing their duty. Example: It was the virtue of altruism which inspired Baba Amte to establish Aanandvan for the welfare of destitute. (iii)Dispassion Definition: Rational and Impartial decision making are the true elements of a person having dispassion.  Significance: Dispassion helps civil servants be upright and not be swayed either in case of conflicts of interest, or political pressure, or ethical dilemmas and help them make the right choice based on the merits of the situation. Example: Religious prejudices may hamper the decision of a civil servant to take impartial decision. Hence, the virtue of dispassion helps her/him to take rational decision. (iv)Objectivity    Definition: Objectivity in governance means adherence to rationality, legality and to proven standards, procedures and norms in institutions by the public authorities. It implies that the governance decisions should be taken based on merit and after rigorous analysis of evidence. Significance: Objectivity is considered one of the most fundamental values in governance because it helps the public authorities to make correct decisions on the merit of evidence. It compliments other values like Integrity, impartiality, non partisanship, empathy, tolerance and compassion in governance. Example: Preparing a Statistical analysis on situation of farmers in India by Swaminathan Committee based on certain parameters and drawing conclusions based on it is Objective evaluation of farmers situation. 2. Given below are two quotations. For each of these, bring out what it means to you in the present context:  (a) “We should never forget that everything Adolph Hitler did in Germany was “legal” and everything the Hungarian freedom fighters did in Hungary was “illegal”.” (150 words) (10) Demand of the question: It expects candidates to highlight the inter linkage between ethics and laws and the possible conflicts which may arise from it. Introduction: Legality or illegality of an action is defined as per the prevalent laws. An action that is legal in one country might be illegal in another, legality, is thus subjective to the place where it is being enforced.  Body: Laws are based on the deliberation, discussion and debate in the society. Violation of law amounts to penalty or punishment that is acceptable to the society at large. Ethics often drive the law and that is why laws are justified and their breach amounts to penalty or punishment that is acceptable to the society at large. However, if the intention behind laws is not ethical, then even though the action will be legal but it will still be unjust as in the case of Hitler. During the times of Hitler, Germany passed laws which circumscribed Jewish citizenship, their means of livelihood and access to legal remedies. It led to social and economic segregation and stigmatisation of an entire community. However, on the international forum it was regarded as gross violation of Human dignity and ethical values of brotherhood, humanity & fraternity. The state-sponsored intimidation, impoverishment and alienation succeeded in driving out about Jewish people and ultimately their  extermination in concentration camps. Likewise, revolutions and uprisings often come up against the state and the laws. These are mostly illegal but not always unethical. In 1956, thousands of Hungarians took to the streets demanding a more democratic political system and freedom from Soviet oppression. These protests were considered as illegal and thus were severely crushed by the Soviets. But one has to understand that the act of these people was not unethical for they were demanding for better welfare measures in a peaceful manner Many social and political reforms in the present day hinge on the factor that, people rose/rebel against the system even if it illegal as per the law. Their rebellion is most of the times is based on the reality of oppression of the citizens/people by the system. The ultimate yardstick for a law should be the ethical underpinnings which it is going to uphold. A law that cannot be justified as ethical under any circumstances must be changed to bring about reforms in the society. Conclusion: Hence, it becomes imperative to make distinction between "just laws" and "unjust laws". Some laws in Germany during the times of Hitler were no laws at all as they were based on wrong notion. Hence, One has not only a legal but a moral responsibility to obey just laws. Conversely, one has a moral responsibility to disobey unjust laws.  2. (b) “Many emotions are products of evolutionary wisdom, which probably has more intelligence than all human minds together.” (150 words) (10) Demand of the question: It expects candidates to write about the fundamentals of emotions. It also expects to draw parallels betw Introduction: Emotions are common to all individuals however, they vary in extent. They are dependent upon thoughts and feelings, physiological changes, expressive behaviours, and inclinations to act. Body: Irrational/ Impartial actions/decisions are based on the well knowledge of their consequences. In this irrational and impartial decision making  emotions play a pivotal role.  Derailed emotions can lead to irrational and sometimes pathological consequences. However, emotions as such are not necessarily irrational. Aristotle, for example, saw anger as a reasonable response to an insult. Importance of emotions in making decisions cannot be emphasized more. Human has developed Artificial intelligence by using its mind. The AI can sure replicate the logical process of the brain but logic alone cannot take the correct decisions involving humans, which clearly indicates the wisdom of emotions. The basic emotions of anger, fear, and sadness made successful leaders from Nelson Mandela to Martin Luther King and many others.  It was Mahatma Gandhi's mastery over emotions of the masses, which gave a climax to the Indian National Movement ultimately lead to independence. Emotions with the evolutionary wisdom have played a prominent role in the great struggles of human kind. For instance, French revolution, which gave the world values of justice equality and fraternity. However, the emotions of more human minds together has put some generations of masses in to the jeopardy. For instance, emotion of anger in to the mob over religious issues has put some of the marginalised communities in to jeopardy in India. Conclusion: Hence, it can be said that, emotions which comes from the masses has made more harm than good. However, the evolutionary wisdom have made the world more beautiful than it was earlier. 3. (a) What do understand by ‘conflict of interests’? How can conflict of interests be managed? Suggest a strategy.  (150 words) (10) Demand of the question: It expects candidates to define conflict of interest. It also expects to write a strategy about handling situation of conflict of interest. Introduction: Conflict of interest situation arises when there is an actual or apparent conflict between public duty and private interest of a public official. In such a situation, an official's private interests could improperly influence the performance of official duties. Body:  Conflict of interest reduces public trust and confidence in the integrity and impartiality of public functionaries. Public servants face conflict of interest due to the nature of their work. The presence of a conflict of interest is independent of the occurrence of impropriety. Therefore, a conflict of interest can be discovered and voluntarily defused before any corruption occurs. Conflict of Interest comes under the principle of Natural Justice and is not codified. As a civil servant, one has to adhere to the standards of civil service values like impartiality, efficiency, integrity and must show exemplary behaviour. Thus, civil servants need to properly handle such conflicting interests and situations. This can be done by adopting strategy as mentioned below, if such a situation arises: Transparency: Declaring one’s conflict of interest to the concerned authorities is the best way. One should rescue him/herself from positions of authority to avoid any conflict of interest. It helps civil servant to come clean and concerned authorities can decide further. Assure integrity: The concerned authority should be assured of integrity and willingness to serve no matter what the decision is made on the declaration. Maintain objectivity: If given the chance to continue working on that case, one should work with objectivity. An open, public data platform enlisting all post-retirement appointments of civil servants will increase transparency. Honourable Supreme Court  in Tansi Land Deal case (2003) has recommended for ‘self imposed discipline’, where persons in public life are expected to maintain high standards of probity. Conclusion: Therefore, the moral obligation on public officials requires that the authority entrusted to them be exercised in the best interest of the people or in ‘public interest’.  Thus conflict of interest is a situation in which personal and professional interest clashes and it might lead to corruption. The best strategy to avoid conflict of interest and its negative outcome is to make a self-disclosure and build the moral fabric of persons in power. 3. (b) What do understand by ‘cooling off period’ in the Indian administrative context? How does it help in reducing corruption? Discuss. (150 words) (10)  Demand of the question: It expects candidates to write in detail about the 'cooling off period' in the Indian Administrative context. It also expects to write about its usability to reduce corruption. Introduction: Officials who retire from sensitive positions are barred from accepting any other appointment for a period of time, normally two years. These cooling-off periods in posts are premised on the snapping off of the nexus between previous incumbency and new appointment by the interposition of a sufficient time gap. Body:  Cooling off period in Indian administrative context refers to the situation in which a person or officer cannot take position of responsibility so as to avoid favouritism and conflict of interest or corruption during his terms of services to the nation. Cooling off period is significant for persons holding high offices as their one decision could be a game changer for the entire country. It is for this reason that certain offices in India are debarred from further employment. This cooling off period is considered more important for judges and other constitutional posts in India because they enjoy people’s trust and their decisions shape public opinion. The rationale behind cooling off period as it helps in reducing corruption is as mentioned below:  Conflict of interest – Cooling off period help to avoid conflict of interest situations. Considering the resource access which one has she/he could misuse them and the power in her/his next job. Ethical dilemmas - Cooling off period is ethical in nature since it allows individual to recollect himself and cool off all grudges and vices. Even if not explicitly mentioned it is in line with the principles of impartiality and non-partisanship. Considering their position, power and authority, they could be an asset for any private company as well as a political party. Curtailing Favouritism: An official can give unbiased decisions as well as take on the government of the day if he knows that his job is secure and that he cannot take job or any position in the near future. Conclusion: Several appointments to administrative bodies require a cooling-off period for individuals so as to eliminate the possibility or suspicion of a conflict of interest or quid pro quo. This cooling-off period must be extended to Indian officials as there is no provision of cooling off period of one year for re-employment of retired Government officials. Q4. (a) What are the changes to public services in the contemporary era? (150 words) (10) Demand of the question: It demands from the candidates to reflect upon the changing nature of public services along with the changing socio-economic and political structures. It is expected to enumerate few changes as the suitable examples. Introduction: The contours of Public services/civil services has transformed keeping in line with the changing nature of demands by the citizens in democratic societies. Public services are considered essential to modern life for moral reasons that their universal provision should be guaranteed. Public service is a service which is provided by government to people either directly (through the public sector) or by financing provision of services. It is the public good i.e. being non-rivalries and non-excludable. As the concept of governance is advancing, it is demanded from public services to become more inclusive, transparent, accountable, outcome oriented and cost-effective, keeping in mind the improved well-being of the people. Body: Public services include healthcare, education, social services, infrastructure like roads and railways, Law enforcement, Waste Management, Environmental protection, Public transportation, Postal Services etc. We can identify various changes to public services. Transparency, accountability, Reducing the discretion, rationalising the procedures, grievance redress, speedy delivery, collaboration with private sector etc. have remained the important areas. Citizen-centric Governance- Good Governance tools like Right To Information, Citizen Charters etc. empower citizens. It ensures that citizens are well-informed. It helps increase accountability and transparency. Citizens get to know their rights. It reduces the discretion on the part of government functionaries.    Right to Public Services- Statutory law that guarantees time-bound delivery of public services. Several states have passed such law. It punishes the public servant who fails to deliver the particular service in stipulated time. E.g. Bihar Loksewaonkaadhikar Adhiniyam, 2011, Maharashtra Right to Public services Ordinance, 2015. Social Audit- It is the accountability initiative where people participate proactively in reviewing the performance various schemes and public services like healthcare. Social audits have been adopted into the implementation process of a number of schemes in India. E.g. Section 17 of the MGNREGA has mandated Social audit of all Works executed under the MGNREGA. After Globalisation, the role of public services in India has changed rapidly. Collaboration with private sector under various PPP models has helped deliver quality public services at optimum level fund utilisation, especially in infrastructure sector. De-monopolisation of public services- e.g. Telecom sector Autonomous PSUs- To bring corporate culture into public sectors to increase the quality of public service. The success story of Delhi Metro Rail Corporation is one such example. It revolutionised the public transportation. E-governance- Computerisation of services, digitisation of records, online access to forms, documentations, billing, and other compliances has reduced physical contact.  It helps easing the access, minimising corruption and harassment of citizens and also saves cost and time. For example, E-choupals in Madhya Pradesh for the benefit of farmers selling their produce, Rural Kiosks/RSDP(Rural Service Delivery Points) in Andhra Pradesh ensures the reach of E-Seva in rural areas. Further use of technology in schemes like Direct Benefit Transfer(DBT), GPS enabled monitoring of projects has helped reduce corruption. Single Window Grievance Redressal mechanisms- Speedy disposal of grievances saves the cost & time and helps citizens through the bureaucratic complexities. Janta Darbar- Another such Grievance redressal mechanism where public representatives or public servants holding important public office allot time slot for citizens to directly meet them and clear their complaints. Code of Conduct- International and National set of code of conducts has evolved over time for various government bodies. It defines the public office and the role of public officials through the ethical and legal-rational frameworks. It codifies maxim of legality and rationality, Maxim of Responsibility and Accountability. That has given the broader meaning to what constitute as Public Services. Conclusion: The quality of public services affects economic growth via its impact on poverty alleviation, human capital formation and corruption. Efficient delivery of public services has direct and indirect bearing upon the performance of the country on Human Development indicators. The content and the contours of public services are constantly evolving, yet there is a long way to go in curbing the corruption and reaching to the last man. 4. (b) Probity is a much broader concept than merely ‘honesty’ or ‘financial rectitude’. Do you agree? Substantiate. (150 words) (10) Demand of the question: The question demands precise understanding of the term ‘probity’. It expects from the candidates to differentiate between Probity and ‘honesty’ or ‘financial rectitude’.  Introduction: Probity refers to the unexceptionable conduct that is honest and sincere. It means adherence to or compliance with the highest principles or ideals and the uprightness of character. It is an unwavering commitment to ethical standards. Body:  Concept of probity is broader, containing in itself the set of virtues. Honesty is the one part of that set. Probity goes beyond avoiding dishonest conduct.  Financial rectitude or financial integrity is essential for probity but, again, not sufficient in itself.  An important yardstick for ensuring probity in governance is absence of corruption. Probity addresses all kinds of corruption even beyond mere financial misappropriation. There are moral virtues and non-moral virtues. Moral virtues constitute compassion, gratitude, conscientiousness, kindness, while non-moral virtues include courage, endurance, self-control etc.  A value such as compassion is good in itself and hence it is an end itself, while values like courage are means to the different end. Here end can be good or bad. Both the subsets are necessary for ensuring probity. But the intention of the act, the behaviour and the end, all of it needs to be good/right to ensure probity in conduct.  The person with probity will take proactive efforts to ensure that ethical standards are adheered to. A person with financial integrity avoids unethical means of gains, unethical/illegal spending.  But a person with probity will try his best to ensure most efficient way of fund utilisation.   Probity covers various important civil services values such as efficiency, transparency, fairness, objectivity, accountability. Second ARC report suggests that adherence to the highest standards of Probity, Integrity and Conduct is a must for civil servant. Conclusion: Probity is one of the most important social construct that needs to be imbibed in society to ensure the members of society are upright. It is particularly important for public office bearers. Laws are there to curb what society considers as unethical conduct. But laws are not enough especially when it comes to behaviour public office bearers. Ex British Prime Minister Benjamin Disraeli said “when men are pure, laws are useless; when men are corrupt, laws are broken”. Probity ensures the highest level conduct. Q5. Given below are two statements. Bring out their significance in the present context. (150 words) (10) (a) “A lack of transparency results in distrust and a deep sense of insecurity.” Demand of the question: It expects the candidates to define the concept of transparency, its importance in today’s context. Also discuss what transparency or lack of it results into. Introduction: In democracy the legitimacy of the state arises from the trust of the people in the state. Erosion of trust results into the weakening of the democratic institutions. And to ensure the trust of the public it is important that government machinery functions in more and more transparent and open manner. Body:  Transparency refers to the openness in the decision making and implementation process. An organization is transparent when its decision-making and mode of working is open to all the stakeholders, public and media scrutiny and public discussion.  A transparent system of administration encourages public participation in government’s decision-making processes. Transparency signifies openness and accountability of administration. Transparency is broadly accepted as a major principle of good governance (World Bank, 2000).  Transparency means “sharing information and acting in an open manner”.In today’s times the role of the people in the decision-making is expanding, as they are becoming more aware of their rights.  It is necessary, keeping in line with the spirit of the democratic values, that all the stakeholders are well informed about the effects of the decisions and the policies that affect them. In fact effectiveness of the decisions increases if they are made following the transparent methods, as stakeholders tend to follow them, as they consider themselves as the part of the process. A lack of transparency essentially hampers the smooth functioning or implementation. Absence of, or inaccessibility to, information often creates a sense of disempowerment, mistrust and frustration. Insecurity breeds from the powerlessness and uncertainty. Transparency is at the centre of how citizens hold their public officials/administration accountable. Arbitrariness on the part of government or discretion in decision-making leaves citizens helpless. This results into distrust and apathy towards public services. Transparency and public participation is necessary because it avoids haphazard policy /decision –making. Public participation in policy matters should be encouraged. Nowadays, proposed Government policies are first placed in draft form on websites and views of stakeholders are elicited. The policy is finalized after taking into account views expressed by the stakeholders.  Transparency is considered to be a key component of public policy and efficiency. The knowledge that decisions and processes are open to public scrutiny can make government bodies work better, by imposing on them a constant discipline. Even the most competent and honest decision-makers need feedback on how policies are working in practice. RTI Act is the foremost measure that government took for promoting transparency in administration.CIC has significantly increased the openness and transparency of the government. It enhances the trust in the system.  Conclusion: It is suggested that government officers need to promote transparency towards (a) citizenry; (b) within their own ranks; and between (c) different administrative branches. They should diligently follow the RTI Act.  In modern times, democracy ensures the government ‘of the people’, while good governance ensures the government ‘for the people’. Transparency incentivises good governance in positive manner. (b) “If a country is to be corruption free and become a nation of beautiful minds, I strongly feel there are three key societal members who can make a difference. They are the father, the mother and the teacher.” (b) “If a country is to be corruption free and become a nation of beautiful minds, I strongly feel there are three key societal members who can make a difference. They are the father, the mother and the teacher.” Demand of the question: Candidate needs to address the importance of the role of the father, the mother and the teacher in cultivating values in individual. Address the importance of the informal leanings along with the formal education. Introduction: This quote is taken from the book Wings of Fire by DR. APJ Abdul Kalam. It seeks to emphasise the importance of the role played by the father, the mother and the teacher in making a virtuous person and ultimately the ethical and happy society, which will be able to curb the menace of corruption and will ensure the wellbeing for all. Body: In early formative years, a child grasps many things by merely seeing. A child acts on what s/he sees. Influence in early formative years remains visible throughout the person’s lifetime. A value a person picks at this stage has deeper impact on developing the behaviour. Children usually consider their parents or teachers to be their role models and try to imitate them. If teachers or parents indulged in bad practices then there are high chances that children pick up the same values and inculcate in them. Teachers and parents must try to radiate positive practices. Family is rightly considered as the primary institute of socialisation, as it imparts the values and the behaviour that influence how person interacts with the other members of society. Parents’ dedication to hard work over sycophancy at office, timely payment of taxes, not jumping the queue to pay electric bills or bribing the temple chief for easy entry are some of the examples by which the member teaches important civic values to his child. A day-to-day life itself becomes the source of ethical education. Learning happens more through observation than dictation method of teaching or the rote method. Hence the character of teacher also impacts the character of a student. It shapes the attitude of the student. Laws are there to curb the corruption but it is not enough if society is corrupt. Precisely Dr.Kalam to bring our attention to the role played by individual agents in weeding out the social menaces. Laws regulate the behaviour from above while virtues regulate the behaviour from within.  Parents and teachers are key players in imparting virtues that make society better. Conclusion: A child’s mind is like clay and parents along with teachers are the potters who can give the shape in a way only they want. ‘Matrudevobhavpitrudevobhav, acharyadevobhava(Respects to mother, father and the teacher, for they are the forms of God.) centuries ago this verse from Taittriya Upanishad tried to emphasise the importance of parents and teachers. It remains relevant even today as rightly expressed in the words of Dr.Kalam to make our nation a better place. Q6.What are the institutional mechanisms available in India to ensure proper utilisation of public funds? Discuss. (150 words) (10) Demand of the question: Question demands to enumerate various institutions and their roles in regulating and scrutinising the fund utilisation.  Introduction: Public fund is meant for public services. Public funds have the potential to transform society, if used properly. Effective utilization of funds for welfare services is one of the key tenets to ensure social and economic justice and meet developmental goals. It includes avoiding under-utilization of funds, over-expenditure leading to fiscal deficit and preventing corruption. Public funds, like any other resource, are scarce and hence their misallocation or misappropriation should be avoided. Body:  Constitutional and statutory bodies are given authority to monitor and check the government spending. There are the mechanisms which make government accountable to the people via legislative bodies.  Institutional mechanisms to ensure proper utilization of public funds: CAG- It audits the government accounts to match receipts and expenditure and see for any kind of misappropriation of funds. Parliamentary Committees- Committees like Public Account Committee or Estimates Committee oversee the effective utilization of funds.  Finance Commission- It divides the total receipts between Centre and states based on rational calculations. FC provides the formula for proper distribution of funds. Central Vigilance Commission- It has been established to address Governmental corruption and ensure public funds are utilized effectively.  Lokpal- It is an anti-corruption authority or body of ombudsman who represents the public interest  Conclusion: No matter how good the policy we frame, its impact drastically depends on the allocation and effective utilization of funds. Therefore, we need strong and independent institutions to ensure the funds are utilized for public welfare. Q7.Don’t you think a legal and not an ethical compliance mechanism for corporate governance is more suited in the India case? Critically comment.(150 words) (10) Demand of the question: The Candidate is expected to differentiate between legal and ethical compliances, bring out the deficiencies in legal framework. Need to critically reflect upon the limitations of legal compliances, also mention the importance of free trade and healthy competition. Introduction: Corporate governance is the system of rules, practices, conventions and processes by which a firm is directed and controlled. Corporate governance essentially involves balancing the interests of a company’s many stakeholders, such as shareholders, senior management executives, customers, suppliers, financiers, the government, and the community at large. Body:  Companies are bodies incorporated under the Companies Act. Companies are set up by entrepreneurs or promoters. They run on the principle of corporate democracy where one share equals one vote. The affairs of the company are managed by a board of directors.  Corporate have profits as the as the prime motive for their functioning. Sometimes it comes in conflict with stakeholders like consumers, shareholders or even community at large. Corporate frauds are usually financial and involve cooking of accounts or ‘creative accounting’. But they also include other types of frauds where statutory laws related to Environmental compliances, labour laws etc. are not followed properly. Frauds mainly happen because of unethical conduct on the part of corporates and also on the part of officers representing regulating agencies. Legal compliance is more suitable in India’s case because of the following reasons: Nepotism in the management- It is common for friends and family of promoters(a uniquely Indian term for founders and controlling shareholders)and management to be appointed as board members. Performance Evaluation of Directors – Although performance evaluation of directors has been part of the existing legal framework in India, evaluation is always a sensitive subject and public disclosures may run counter-productive. Accountability to Stakeholders – Various general duties have been imposed on all directors, directors including independent directors have been complacent due to lack of enforcement action Role of the independent directors is of serious concern in corporate governance. Independent directors appointed by the promoters may not take the interests of the small investors into account. Removal of Independent Directors – In India there are instances of independent directors not siding with promoter decisions have not been taken well and they were removed from their position by promoters. Executive Compensation – It has been identified on many instances where executives keep on withdrawing high salaries and other fees even if company is not doing well.  Executive compensation is a contentious issue especially when subject to shareholder accountability. Risk Management – Indian companies certainly don’t have a clear idea about the risk management and predictions. There is the provision for Risk Management Committee (RMC).  In IL&FS crisis that RMC meetings were not conducted even after accounts were showing negative signals for many quarters. The accounts can be slightly manipulated to show higher profits. Such manipulation tends to hamper small investors more who are unaware about the internal functioning. Non-compliance- Despite several regulations in place, we often see non-compliance of those by the companies. Revolving Door- A revolving door is the movement of high-level employees from public-sector jobs to private sector jobs and vice versa. It hampers the regulatory oversight. To address some of these issues The Companies Act, 2013 has been enacted.: Some provisions have been laid down under the Securities and Exchange Board of India(Listing Obligations and Disclosure Requirements) Regulations, 2015,forenhancing transparency in transactions or dealings of a company. Penalties have also been put in place under the Companies Act and SEBI LODR, thereby ensuring compliance with corporate governance provisions a mandatory affair. Ethical compliance, however, is required too as without having the required ethical framework or value system in a person or organization, we cannot expect the laws to be followed, as given the legal expertise companies can hire, there can always be away around the existing laws. Also, we have seen, many companies did have ethical governance before enactment of the law. India requires a robust legal mechanism to avoid lapses in corporate governance while having ethically correct members on board of corporations.  Also, the recommendations of various committees like UdayKotak Panel, Narayana Murthycommittee and Kumarmangalam Birla committee needs to be followed. Conclusion: Laws are the regulation from above, while Ethics refers to the regulation from within. In case of corporate governance we should encourage the latter one. But as the prime motive from within remains profit-making, regulatory bodies are required to legally put check on their wrongdoings. Yet government should avoid old days of license Raj where regulatory bodies had entered deeper into the day-to-day functioning of the corporate. It is necessary that laws are made with clear and simple provisions with no scope for getting around them. Q8.Define attitude and expand upon the tri-component model of attitudes.(150 words) (10) Demand of the question: Question expects candidates to explain the concept of attitude and how the three components viz. Cognitive, affective and behavioural component affects shaping of the attitude. Introduction: Attitude refers to the mental make-up of the person towards a particular object. It may be positive or negative. It may change from person to person for a particular object. Attitude is subjective individual interpretation of objective outside world. Object includes people, things, events, and issues.  Attitudes reflect in the feelings and the beliefs of persons and that shape their behaviour.  Body:  The three components that shape attitude are namely the Cognitive, the Affective and the Behavioural component (CAB). Cognitive Component-  This is based on the available information/facts one has about the particular object. It helps form a judgement or an opinion on the object. It can be favourable or unfavourable opinion. Information can be true or false but it shapes the attitude accordingly. Affective component : This component is related to the emotions. The emotions like fear, sympathy, hatred, pleasure etc. about the attitude object influences the formation of the attitude. At times it can even cloud cognitive component. It is very strong force to change the attitude and necessarily to change the behaviour of a person. Behavioural component: This component is shaped by the first two components. Cognitive and Affective component create a predisposition/prejudice towards particular attitude object for acting in a particular way. The behavioural component consists of the tendency of a person to behave in a particular manner towards an object.  Out of the three components of attitudes, only the behavioural component can be directly observed. One cannot see another person’s beliefs (the Cognitive Component) and his feelings (the Affective component). These two components can only be inferred.  Conclusion: Our attitude influences our actions. It helps shape our goals. It ensures how we perceive, view and approach the different aspects of life. It shapes one’s work culture, interpersonal relationships, and political opinions. Attitude remains the most influencing factor in shaping the personality. SECTION – B Q9. Your younger brother has got influenced by a radical religious outfit that promotes hatred for the minority and aspires to build a nation based not on territoriality but on religion. You have tried to make him understand the fallouts of such thinking and even reprimanded him for joining such groups. However, the conviction with which he debates and counters your arguments has forced you to rethink about your own notions. He presents gory details of the atrocities committed in the past by the minority community and also gives an account of the appeasement policy pursued by various governments since independence. He argues as to why can’t the majority assert its identity in its own country. He seems perturbed by the fiction that the minority community is trying to outnumber the majority in the near future by exploding a population bomb and also through forced conversions. You get to realise that even though his fears are real, their basis is rather absurd and illogical. In a situation like this, how would you convince your brother to abandon the path of radicalism? (250 Words) (20) Demand of the question: It expects candidates to first spell out the number of stakeholders involved and effect on them  along with explaining the validation/invalidation of the arguments put forth by brother. It also expects to mention the way in which you will convince your brother to abandon the path of radicalism.   Introduction: Radicalism is set of beliefs or actions of people who advocate thorough or complete political or social reform. Radicalisation refers to the process of an individual’s transformation from a moderate, law-abiding citizen into an active, anti-state, violent extremist. Body: The urge to take revenge on the present innocent people for the sins committed by ancestors along with Politicization of religion have led some groups to chose the path of radicalism. The aforesaid situation is complex and involves the following stakeholders: Brother Minority Community Majority Community Radical religious outfit Myself  Secular fabric of Nation The arguments put forth by brother are valid to an extent. However, blindly following them amounts to culmination in to radicalisation.  Hence, it becomes my moral as well as social responsibility to convince him about how absurd and illogical  his notions are and why he should do away with such kind of notions. I will present following arguments to convince my brother to abandon path of radicalism. I will explain him how all religions have similar teachings and all show a common path for life. I will elaborate him in detail about how some miscreants teach some wrong teachings to the followers and misguide them. For instance, Easter Sunday bombing on church in Sri Lanka happened due to this kind of wrongful teaching.  I will tell  him the long standing rich history of India, how various communities peacefully co-existed and the rift came only to gain some benefits.  Earlier it was done by some rulers, then by Britishers and now by certain groups. People are however the same people of our land sharing a common ancestry. Unity in Diversity: Our constitution as well as the spirit of our nation lies in Unity in diversity. I will aware him about the rich heritage our culture and tradition. How they are intermingled and how they deal with each other by practising tolerance. This rich tradition can only be preserved if we all stay peacefully together despite our differences. Issue of population explosion: Being a population bomb might increase number of minorities, but it does not provide them with an equal social standing in terms of social or economic or political influence which still majority holds. I will also  show him the abject poverty, ghettoisation, deteriorating health conditions which minority faces. I will tell him that not all religious conversions are forced conversion. Some people opt on their own will to admire and adapt other religion. Hence, we can't look at all the religious conversions through same glass.  However, In case of forced conversions, there are legal measures and those are being enforced in any case that comes to light. Role of politics:  Minority appeasement has been done by political parties in the past, but they were directed towards votes. Even after the appeasement social status of minorities is way below the majority and even politically, it is majority that holds influence. Conclusion: While addressing all his absurd and illogical arguments I will reason them on above mentioned grounds. I will also take help of my family members, religious scholars, elders and friends in our locality to help him to understand the grim reality of the situation.  True knowledge is real power, hence there is a need to de-radicalise these kind of youths, so that their true potential can be utilized to build a truly tolerant society which lives in peace and harmony.  Q10.You are posted as the block development officer in a district. Menstruation is a taboo in one of the villages of the district and people hardly talk about it. In fact, menstruating women aren’t even allowed to enter temples and are forced to sleep on the floor. Women use clothes as they can’t afford sanitary napkins or are too shy to buy it from the general store. When a renowned NGO attempted to spread awareness towards hygiene during menstruation by talking to the local women, the villagers created ruckus and forced the representatives of the NGO to leave. Further attempts by the NGO to approach the women was thwarted by the Gram Panchayat. The women of the village have been warned by their family members to stay away from such discussions and maintain their dignity.  The NGO brings the issue to your notice and asks for your support. Now answer the following questions: (i)What in your opinion is  the main reason behind villagers protesting?  (ii)What steps would you take to address the issue? (250 Words) (20) Demand of the question: It expects candidates to address this issue on two grounds. First one is, it expects to opine about what might be the reason behind villagers protesting. Second one being it expects to write about what steps you as a block development officer will take to address the issue.  Introduction: Menstruation is a phenomenon unique to girls. However, it has always been surrounded by taboos and myths that exclude women from many aspects of socio-cultural life. Following are the main stakeholders in this issue: Block Development Officer (BDO) Gram Panchayat Villagers (Includes both Male and Female) NGO Body: (i)The main reason behind villagers protesting:  Taboos about menstruation present in many societies impact on girls’ and women's emotional state, mentality and lifestyle and most importantly, health. It can't be certainly said that there is only one reason due to which villagers are protesting. A  combination of specific reasons can be attributed  behind protest of villagers: Role of Patriarchy: System of patriarchy has mostly impacted all aspects of society. It has also impacted the women lives in general and menstruation related taboos in particular.  Cultural norms and religious taboos on menstruation are often compounded by traditional associations with evil spirits, shame and embarrassment surrounding sexual reproduction. It might also be believed that restricting certain kind of food for  a women while she is menstruating is essential to avoid disturbance or termination of the menstrual flow. In some societies there is perceptions on notions of purity and pollution. Bodily excretions are believed to be polluting, as are the bodies when producing them. All women, regardless of their social caste, incur pollution through the bodily processes of menstruation and childbirth. Men and boys typically know even less, but it is important for them to understand menstruation so they can support their wives, daughters, mothers, students, employees, and peers. Sensitization of health workers, Accredited Social Health Activists and Anganwadi workers through NGO regarding menstruation biology can  be done so that they can further disseminate this knowledge in the community and mobilize social support against busting menstruation related myths. (ii) Steps  to address the issue: The challenge, of addressing the socio-cultural taboos and beliefs in menstruation, is further compounded by the low girls’ knowledge levels and understandings of puberty, menstruation, and reproductive health. Thus, there is the need to follow a strategic approach in combating these issues as mentioned below: The first and foremost strategy in this regard is raising the awareness among the adolescent girls related to menstrual health and hygiene. Community based health education campaigns could prove worthwhile in achieving this task. There is also need to spread awareness among the school teachers regarding menstruation. It will help to address the doubts and questions of villagers. As a BDO it become responsibility to meeting the gram panchayat members and  convince them. Take them into confidence and involve them in the issue. In case they are male then try to take help from women members of their family. Low cost sanitary napkins can be locally made and distributed particularly in the village locality. For instance, Government of India has approved a scheme to improve menstrual hygiene for 1.5 Crore adolescent girls by distributing low cost sanitary napkins in rural areas under the National Rural Health Mission since 2010. Conclusion: With this kind of Multi-dimensional approach we can remove out the taboo from the minds of villagers. Menstruation is nothing but a very normal biological phenomenon, and adolescent girls and women should be helped to understand that they have the power of procreation only because of this virtue. Once this is this done, it will not just help a woman to have an empowered life but it will also help to have well developed society too.  Q11. You have just been posted as the SP of a district known for its rich mineral resources. However, the abundance of natural resources also invites illicit trade and associated crimes. On the very first day of your induction, you are told by your superiors that the local mining mafia is really powerful there and you must be careful in your dealings with them. As disturbing the scenario in the district might be, you are more perturbed by the fact that no one seems to come up with a solution. You start getting the sense that the wrongdoings of the mining mafia largely go unnoticed as many senior bureaucrats and politicians are hand in gloves with them. The mafia enjoys heavy clout in the political circles as they are the prime source of electoral funding for the political parties. Similarly, bureaucrats enjoy their cut in the money that is extorted by the mafia from various stakeholders. Government regulations and institutional control don’t seem to exist in your district. Environmental norms are flouted, innocent and honest people are targeted if they don’t comply and crime is on the rise as the nexus of lawlessness and money power breeds criminals.  Being the determined officer you are, you start tightening the noose around the mafia. You start streamlining the systems hitherto kept in abeyance by your predecessors. You start a consultation process with the local landholders whose rights have been compromised to get a close view of the ground reality. You get to know that the youth in your district is largely jobless and gets infatuated by the money on offer by the mafia and take to criminal activities easily. Hardly any of the miners possess a valid license to operate and yet mining activities continue unabated. Immediately, you start a closure drive on the illegal mines and within a week, only licensed and authorised mine operators are running the mines in your district. You also convince the youth to stay away from criminal activities and rather participate constructively in the drive launched by the police to make the district a safer and better place. The local newspapers are praising your courage and dedication. There doesn’t seem to be a problem and you start wondering- why was it so difficult in the past? However, within a week, you and your family start receiving death threats from the mafia. Calls start pouring in from senior bureaucrats and politicians to turn a blind eye to the situation in your district. In fact, your office stops receiving the monthly allowance to maintain the police force. You are forced to spend from your pocket to fund the diesel that goes into patrolling the disturbed areas of your district. The situation turns worse when your salary is stopped for a frivolous reason. All of a sudden, you find yourself alone. Everyone is against you- the colleagues in the department, the political masters and of course the mafia. You start thinking about the safety of your family and send them to your parents in your home state. All your good work has been rewarded with threats and fear.  Now, answer the following questions:  (i) Does there remain any motivation for you to work for the betterment of the society? (ii) Don’t you think such things happen in reality? How do you suggest one deals with  it? (iii) Can a person single handily change a system? Critically analyse. (250 Words) (20) Demand of the question: The case study is well elaborative and Candidates needs to identify various stakeholders and their interests.  Also candidates need to identify the ethical values involved along with the proper role expected to be played by the officer according to the rulebook in letter and spirit. Introduction: Stakeholders involved in this case study Myself (SP of the district) My family Mafia - Criminal Elements of Society Politicians Bureaucrats in nexus with Mafia Land owners Jobless Youth Miners- legal and illegal Colleagues in the department Media District Community Values Involved here are:  Courage Integrity of an Officer Leadership Emotional Intelligence Body: The given situation is very critical one and candidate needs to address following questions keeping in mind the said situation. (i) Does there remain any motivation for you to work for the betterment of the society? The prime motive for honest Public Servants remains the core conscience of a person to serve the society. They are neither driven by the external rewards as the primary source of motivation nor are they discouraged by the obstacles in their path. If it is the external source of motivation that drives a person then in such critical situations one is bound to get de-motivated. In fact throughout the life of the public servant there comes many instances that they can be forced to quit or accept wrongdoings. One will be able to take stand only when one has the clear purpose of betterment of society in mind and unabated commitment to the ethical framework prescribed for the civil servants. If a person lacks such a sense of purpose and commitment to the values then s/he will succumb to the notions that system itself is corrupted and cannot be changed. Civil Servants should realize that the primary duty of civil servants is to serve without expecting anything in return but with a vision to build a stronger India. A civil servant needs to recollect his Oath of Office which says that one has to abide by Constitution without fear or favour. Integrity of a civil servant must not be related to the motivation or demotivation that comes from the day-to-day happenings. (ii). Don’t you think such things happen in reality? How do you suggest one deals with it? Yes, there are many instances to suggest that such things happen in reality. It puts the career of the civil servant in jeopardy. We hear the news often that many upright officers face the brunt of the politicians and get transferred frequently. Sincere officers get many hurdles in carrying the day-to-day job. Yet the civil servant also has the protection from these things. Indian constitution and laws have provided several ways to deal with such situations to protect honest civil servants: Administrative tribunals: These are specifically set up to deal with issues faced by civil servants. Judiciary: There are judicial courts till Supreme Court wherein a person can approach to fight for right cause and get justice. Public support: Gather enough evidence, and take support of public to bring out the issues being faced in the region to strengthen you case. Seniors: Take help of seniors who have handled such cases before and take their advice on how to deal such issues in future too. Civil Society - Activists and NGO’s: Since environmental norms are being floated, there are several regional, national and international organizations fighting for environmental protection, it can be brought to their notice which will help make it a national issue and put the pressure of public opinion in one’s favour. Media: Media is again a strong tool to shape the public opinion by sharing the facts on the ground.  Honest officer should always keep in mind that there is no way of using wrong means for the right ends. Means and ends both needs to right. A public servant should adhere to all legal rules and regulations, and maintain procedural integrity, particularly in such critical conditions. (iii). Can a person single handily change a system? Critically analyse A public servant should not consider himself/herself as the sole torchbearer of honesty and integrity. There are many honest public servants are working in the system. In given condition everyone seems to be going against the one honest officer. Here he will have to hold on to his ethical stand. It is certainly possible to single handily change the system. History is the proof to show that nothing is impossible if a person is determined and focused. He can single handily bring about a change. If everyone thinks that the system cannot be changed, then the society would not evolve & progress. It would stagnate and deteriorate. Every change in society will start with some or the other person. That single person is able to persuade &convince the fellow community members to rectify their wrong beliefs/actions. Thus, one needs to have a positive frame of mind and a conviction to bring about the change in system for the betterment of people, even if one is alone during the initial days of difficult journey. Conclusion: One needs to believe that if there is truth in our beliefs/actions people will eventually recognize the efforts and the system will change. The motto of our emblem “SatyamevaJayate” inspires us to continue our fight to change the system. You can give examples of Gandhiji, Ambedkar, JyotibaPhule, Basavanna, Adi Shakaracharya, Martin Luther King, Rosa Parks, Aung San SuuKyi etc. who were all determined to bring changes in the system especially when at times they were lonely in their fight. (It should be noted that above mentioned historic personalities were fighting the system which was absolutely alien to the people and which was highly exploitative. In given case study the concerned SP has the right procedural means to reform the system. Hence he should adhere to. It is difficult to cope up with the given condition, but He has legal authority and constitutionally assigned duty. Here it is correct talk in terms of reforming the given system to make it better rather than changing the system altogether.) Q12. As an agent of the secret service of the armed forces, you have the responsibility of spying over the enemies and pass on vital strategic information to the government of your country. You have been trained and nurtured in a manner that you won’t even hesitate to kill an innocent person if he/ she becomes an obstacle in your duty even unknowingly. For you, your country comes first and you regard your duty as the foremost service one can render to one’s motherland.  One one occasion, you intercept a message that the enemy country is planning to bomb the pilgrimage base camps stationed at various locations in the bordering state. In fact, troops have been mobilised to give effect to the plan and it is a matter of hours that heavy bombings would start. It is not even possible to launch a counter offensive at such a short notice as major battalions of the army have already been sent to provide relief in the flood hit areas of the country. Waiting for the army to take command would mean severe loss of lives in the base camps.  When your commanding officer comes to know about the scenario, he chalks out a plan and deputes  you to give shape to it. However, you are shocked to know his plans. In order to avoid the bombings, he wants you to immediately bomb a school in the enemy territory which lies in close proximity to the border. By doing this, the attention of the enemy would get diverted to the school and they might divert their troops to the school for immediate relief. This would buy some time for the armed forces of your country and they would be able to evacuate the base camps and also take positions to retaliate in case of eventual bombings.  What options do you have in this situation. Which one would you choose and why? Examine the pros and cons of each of the alternatives. (250 Words) (20) Demand of the question: Candidates are supposed to elaborate on all the alternative options and the core ethical dilemma before stating what option one should choose Introduction: Given case provides a scenario where a secret service agent who is likely to face the sharp ethical dilemma in line of his/her duty. While profession demands the completion of the committed duty i.e. protecting the citizens of one’s own country, the basic humanity in person is likely to pose a question, ‘at what cost?’ Why innocent children and their parents have to be victimised? Body: Agent’s inaction would cost lives of innocent civilians while the action suggested by the commanding officer also puts lives of innocent civilians of another nation in jeopardy. There seems a clear conflict between his professional commitment and a personal value. Stakeholders involved: Me as agent of secret service Pilgrims Children Commanding officer Enemy troops Options available to the agent: Follow order of commanding officer and bomb a school. Pros:  It would give valuable time for army to protect pilgrims. One’s duty towards nation is performed. Acting according to the order of commanding officer. Cons: Loss of lives of children. Negative image of nation. Likely to increase hatred against nation and chances of retaliation. Inner voice blaming the ‘self’. Disobey commanding officer with regard to bombing school and take no action. Pros:  Innocent children saved. No self-guilt. Cons: Failure to perform one’s duty towards nation- not protecting people of his country. Likely to pose questions over agent’s work credentials in future. Refuse to bomb a school but take action with available force. Alert local police forces about the threat and take every possible measure to protect pilgrims. Pros: Innocent children saved and thus no self-guilt. Sense of performing one’s duty. Cons:  Uncertainty over results that is likely to happen.  Not just pilgrims but it also puts involved personnel at risk. Refusing to obey orders of commanding officer.  Likely to pose questions over agent’s work credentials. Refuse to bomb school but try convincing commanding officer on modifying plan to attack on strategic location or other areas causing lesser casualty and thus achieving objective of diverting enemy troops. Pros: Saves children at the same time gives time to army to protect pilgrims.  One’s duty is performed. Cons: Uncertainty on whether enemy troops will divert their action.  Officer may not agree on modifying plan. Best option in this scenario would be option 4. Though option 3, much like option 4 helps overcome professional duty vs personal value dilemma, but it risks lives of both pilgrims and locals force involved. And there is also uncertainty that bombing school will divert actions of enemy troops. They might still proceed with their plan. So when our plan of action is based on probable outcome, it would be better to carry out bombing on strategic location or other infrastructure rather than on schools. This might give time for armed forces to save pilgrims. Conclusion: To conclude one need to express that all aspects are uncertain as far as possibilities of outcomes is considered. Hence one needs to choose the option that is likely to result into the least possible casualties. Q 13. It is said that a doctor performs the noblest profession. Doctors are considered God by many in different parts of the world. You were always attracted to this profession and wanted to become the best doctor in your area so that you also could save lives and feel good about it. You were lucky that your parents supported your aspirations. With their support and your hard work, you cracked the medical entrance examination, attended one of the most reputed medical colleges in the country and finally became a doctor. You joined a government hospital and it was nothing less than dream come true for you. You were ready to serve the people with all those life saving skills that you had learnt in the college. Meanwhile, you got married to a beautiful girl and had two lovely kids with her.  Work- life balance was going smooth until one day, the news of a new viral strain spreading in the southern most coastal state of the country broke out. The strain was so severe that even many doctors and nurses had succumbed to the infection and died. The situation had become so worse that no doctor was ready to treat the dying patients.  This troubled you a lot and you felt that the time had come to serve the people in the truest sense. You decided that you would visit the state and render your services to the ailing patients. However, your parents, wife and children are strictly against this decision. They fear that even you would get affected by the virus. You tried to convince them by telling them that the foremost duty of a doctor was to save lives no matter what and if you didn’t go, the guilt of turning your back towards the needy would haunt you forever. But your family members aren't ready to let you go. After all, you are the sole breadwinner and caretaker of the family. How can they simply let you be in danger? They love you so much and can’t afford to lose you.  Now, you were in a complex situation that demanded a reasoned yet moral action from you. What would you have done in this situation? Examine the alternatives available to you and comment on the pros and cons of each one of them. Also, which alternative you would have chosen and why? (250 Words) (25) Demand of the question: It expects candidates to look for all probable options which he/she can take. It tests decision making capability of the candidate in emergency  situation. It also expects that candidate should take decision only after considering all probable options.  Introduction: Personal and professional values often conflict each other for public servants where they have a responsibilities towards the committed duty on the one hand and responsibilities as the family member on the other. The ethicaldilemma that emerges out of the conflicting situation is difficult to resolve especially in contemporary age which requires dynamism. Body: Stakeholders involved in the given scenario- Me as a doctor, Son and Husband My family People of the country Government Ailing patient Issues- Duty as a doctor vs. Responsibility of relationships – I became doctor in the first place because I want to serve the society but my family is also dependent on me since I am the sole bread winner of the house.  Personal values vs. Professional values – Love and duty are the two horses here pulling in opposite direction.  Ethics of self-interest vs. deontology – Fulfilling interest of my family or doing the right thing, similar to the way many soldiers go to border despite having family. Crisis of conscience – I have always thought that the duty of a doctor is the best profession because it can save lives, now when the situation demand show can I run from it. There goes the saying that “Duty is the death of love”. But being doctor is a kind of a profession where the value of the love is required to be adopted while performing the duty beyond loving one’s own family.  Options available Do as the family says: I have always dreamt to be a doctor but that was before I was married. After marriage I am also a father. Moreover, my parents are old and dependent on me now, if something happens to me then there is no one to take care of them. Pros: It will save my life I can take care of my family Fulfilling personal duties Cons: Running away from my professional duties Lack of courage to stand for the values I believe in Regret for the whole life Go to the location and serve patients: I started this medical journey because I wanted to serve people, now when the time is there I cannot run away. It is like a soldier on duty, he cannot run away when his country needs him, especially during war.  This stand can be also supported by the principles of utilitarianism as well as deontology, as by treating those patients I will be serving humanity as well as fulfilling my duties. Pros: Serving the humanity Avoiding the crisis of conscience Fulfilment of The Hippocratic Oath taken Cons: Fulfilling my duties I might get infected Injustice to my family Violating personal duties Take the help of government and like-minded individuals I can take help of government in this case, since no one is willing to go and I am ready to risk my life. I can ask for the following aid from the government. a) Detailed information about the virus, how its preads what are the precautions I should take etc. b) Support staff as well as any individual willing to go with me. c) The government can make an appeal to public so that individuals can come forward. d) Financial support or a job to one of my family members in case I die. Pros:  Fulfilling my personal and professional duties Life security of my family Showing courage at the time of hour Cons:  High possibilities that I might die Emotional trauma to the family Government support might be nil Starting a social media campaign to support me I can take help of social media, NGOs to get like-minded individuals and to know more about the virus before jumping in. I will take an informed decision as to how will I go about it because simply going and losing my life won’t do any good, if I can find a cure or at least help in the research, it will help humanity a lot. I can also ask for crowd-funding to support my family if my insurance amount is not proper. Getting public support will also help me in convincing my family and changing their attitude. Pros:  Public awareness I can inspire others Cons: I might not get support Emotional trauma to family My solution As Buddha advises that SamyakMarg is the best way out of a dilemma, moreover the principle of golden mean also says that wisdom lies in choosing between two extremes. Since going there without preparation amounts to foolishness and I might infect others too. It is important to gather information and take government help before going to serve the patients. The research will also help me in convincing my family as well as other doctors to join me. The support of government and public willalso give a boost to my family. It is my duty to serve people especially during this time of crisis and I cannot run away from it. Getting insurance for my family and arranging for the financial needs will help me in fulfilling the personal duties and responsibilities as well. Conclusion: One cannot shy away from his duties owing to the personal reasons as it is akin to cowardice. I have chosen this profession to serve patients and nation, not doing that will not be fair to me personally and to the principles I hold dear. However it doesn’t mean that one should completely ignore the personal duties. An emotionally intelligent person will try to find a balance between both rather than choosing one over the other. Q14. You are the founding member of a political party that was born out of an agitation against corruption in the country. Even though you and your partners never wanted to contest elections, the circumstances were such that you were forced to enter the very same political system that you abhorred for its disconnect and corruption. But then, someone has to cleanse the system after all.  Your decision to form a political party has been welcomed by many and your supporters are eagerly waiting to witness the transformation that your party would bring about in the political discourse. However, as the days pass by, reality dawns at you and your partners. It is getting difficult to run the party and pursue its agenda without adequate money. Donations have thinned and the party workers are demotivated. A sense of despondency has creeped in wherein everyone believes that it is impossible to bring about a change without money and muscle in Indian politics.  While you are deeply worried about the party and its followers, a business tycoon accompanied by a  famous media honcho visits your party office. They have an offer. If you agree to twist the tender and procurement rules in the favour of the businessman after you come to power, they are ready to fund your party and become a mouthpiece of your ideology and agenda through various media.  This appears to be an offer that you can’t resist. What are you going to lose? Literally nothing! Rather a symbiotic relationship with the gentlemen would ensure the revival of your party and also a genuine shot at power. Your partners and party workers are infatuated by this offer and they insist that you accept it. Even though it would mean a compromise with the fundamental ideals of your party, sometimes stepping down from the moral plank can give you enough leeway to fulfil your aspirations.  Now answer the following questions: (i) What options do you have in this situation? (ii)Of all the options available to you, which one will you choose and why? Elaborate. (250 Words) (25) Demand of the question: These kinds of questions demand to test the accurate decision making capability of candiadate according to situation. Hence, this questions demands that candidate has to take decision while balancing his/her fundamentals along with reality of situation.  Introduction: Probity and integrity are the most important characteristics which define the character of an individual. It is about adhering to best moral principles even in the worst of the times. It is about doing what you believe in, to be consistent in what you believe in, and in what you say and what you do. Political integrity is even more difficult because the lust of power tends to make people corrupt in various forms. Politicians are tempted to adopt an unethical path. The given case study is also about the tussle, an easy path to power or to struggle with your values and ideals. Body: Since the origin my political career is from an agitation against corruption and the offer given by the businessman is nothing short of a corruption, accepting the offer is like turning my ideals upside down. Ethics and morality are a set of standards which one follow and choose the rights and wrong. It is the moral values and ideals which generates the trust. In politics money can give short-term shot to power but in the Long-run ideology and strong fundamentals survive. The kind of corruption in the form of quid pro quo suggested in the given case has been at the heart of the dis-functioning, non-responsive democracy. It hampers the spirit of democracy and betrays the faith of the common people in the democratic procedures. Money power overwhelms the free and fair functioning of our democracy where ‘one-person-one-vote’ loses its meaning in front of ‘one-rupee-one-vote.’ (i)Options available to me: 1.Accept the offer –  Practically it will help my party since we are short offunds and the motivation of party workers is also dying. It is gettingdifficult to run the party and if there is no party left who will spread ourideals. However it means moving down from the committed principles. Pros: Funds for the party that will ensure its survival Motivation to workers Can fight elections against the well-established parties Cons: Compromising my ideals Moving on the slippery slope Losing trust of public 2. Reject the offer-  We started the party to fight against the corruption;moreover the real intention was never to start a party but to fight corruption. Accepting the businessman’s offer is like throwing the movement down the drain and becoming just another party to the corruption. It is like moving down a slippery slope where I am justifying the wrong thing. Pros: Maintaining integrity Trust of public Beneficial in the long run Cons:  Workers might be demotivated  Party might not survive  Will not be able to fight elections 3.Reject the offer and approach public to get funding, increase social media campaigning and asking individuals to join the revolution. As compromising with the ideas will degrade the image of party in public and thus in longer run it might not even survive. Not accepting the corrupt money might slow down the movement but it will help us in the longer run. Pros:  Making public aware about the ideals Gaining public trust Maintaining integrity Cons: Delay in coming to power Party workers might leave the party Delay in fighting corruption (ii) Of all the options available to me, as discussed above, I will choose the third option because compromising with the ideals as well as the funds is wrong for the party and the movement. I cannot accept the offer because it is like a deal with the devil, moreover it is a betrayal to the trust of many who joined the movement and have shown faith that we are going to do something good to the nation. Accepting the money can boost the party in the shorter run but in the long term it is going to tarnish the image, as I will also be seen as a corrupt and opportunist who just wish to get power.  As Gandhiji during India’s freedom struggle said to the congressman when they were eager to enter politics and assume power after Government of India Act 1935, that you should hold the chair lightly and not tightly. He advised them not to compromise with their ideals and not to fall into the vicious trap of power. India has changed but the political ideals are still the same. Conclusion: Politics is a game of power but playing it dirty serves no purpose as it is against the basic principle of serving the masses. Politics is for welfare of masses where power has to be used for service not to rule. Compromising with the ideals and ideology means that the party is not based on ideals and ethics but it is based on opportunistic tendencies where the party can do anything wrong to come to power. As rightly put, Politics without principles is a sin. Not merely the ends but means to the right end also needs to be right. TLP HOT Synopsis_FULLMOCK_4 PDF

Daily Prelims CA Quiz

UPSC Quiz - 2020 : IASbaba's Daily Current Affairs Quiz 8th Oct 2020

For Previous Daily Quiz (ARCHIVES) - CLICK HERE The Current Affairs questions are based on sources like ‘The Hindu’, ‘Indian Express’ and ‘PIB’, which are very important sources for UPSC Prelims Exam. The questions are focused on both the concepts and facts. The topics covered here are generally different from what is being covered under ‘Daily Current Affairs/Daily News Analysis (DNA) and Daily Static Quiz’ to avoid duplication. The questions would be published from Monday to Saturday before 2 PM. One should not spend more than 10 minutes on this initiative. This is a part of our recently launched, NEW INITIATIVE IASbaba’s INTEGRATED REVISION PLAN (IRP) 2020 – Road Map for the next 100 Days! FREE INITIATIVE! We will make sure, in the next 4 months not a single day is wasted. All your energies are channelized in the right direction. Trust us! This will make a huge difference in your results this time, provided that you follow this plan sincerely every day without fail. Gear up and Make the Best Use of this initiative. Do remember that, “the difference between Ordinary and EXTRA-Ordinary is PRACTICE!!” To Know More about the Initiative -> CLICK HERE SCHEDULE/DETAILED PLAN – > CLICK HERE Important Note: Don't forget to post your marks in the comment section. Also, let us know if you enjoyed today's test :) After completing the 5 questions, click on 'View Questions' to check your score, time taken and solutions. To take the Test - Click Here